Vous êtes sur la page 1sur 147

15

LECTURES IN ANAESTHESIOLOGY
FOR
MEDICAL STUDENTS

BY
PROF. BRIG. M. SALIM SI(M)
MBBS: MCPS (Pak); D.A. (London); FFARCSI (Dublin)
FRCA (London); FCPS (Pak); Ph.D ,FRCP; FICS, FACS.
Diploma in Acupuncture (China); D.Sc. (Hony)
Fellow Medicina Alternativa.

HIGHER EDUCATION COMMISSION


ISLAMABAD

Copyrights @ Higher Education Commission


Islamabad
Lahore Karachi

Peshawar

All rights are reserved. No part of this publication may be reproduced, or transmitted, in any
form or by any means including, but not limited to, electronic, mechanical, photocopying,
recording, or, otherwise or used for any commercial purpose what so ever without the prior written
permission of the publisher and, if publisher considers necessary, formal license agreement with
publisher may be executed.
Project: Monograph and Textbook Writing Scheme aims to develop a culture of writing and to
develop authorship cadre among teaching and researcher community of higher education
institutions in the country. For information please visit: www.hec.gov.pk
HEC Cataloging in Publication (CIP Data):
Salim, M.
Basics of pain medicine
1. Pain
2. Medicine
Includes index
616.849-dc22
ISBN: 969-8963-00-6
First Edition 2007 (Published by HEC)
Second Edition 2014
Copies Printed: 500
Published By: D. G. Administration, Higher Education Commission, IslamabadPakistan
Disclaimer: The publisher has used its best efforts for this publication through a rigorous system
of evaluation and quality standards, but does not assume, and hereby disclaims, any liability to
any person for any loss or damage caused by the errors or omissions in this publication, whether
such errors or emissions result from negligence, accident, or any other cause.

15

LECTURES IN ANAESTHESIOLOGY
FOR
MEDICAL STUDENTS

BY
PROF. BRIG. M. SALIM SI(M)
MBBS: MCPS (Pak); D.A. (London); FFARCSI (Dublin)
FRCA (London); FCPS (Pak); Ph.D ,FRCP; FICS, FACS.
Diploma in Acupuncture (China); D.Sc. (Hony)
Fellow Medicina Alternativa.

Professor of Anaesthesiology & Pain Medicine


Islamic International Medical College, Rawalpindi.

Honorary Consultant & Instructor


Armed Forces Post Graduate Medical Institute, Rawalpindi.

President: Society for Treatment and Study of Pain (STSP)

Chief Editor, JIIMC

Chief Editor, Anaesthesia, Pain & Intensive Care

Patron,, Rawal Medical Journal

FORMERLY:

Prof. of Anaesthesia, Rawalpindi Medical College


Holy Family Hospital, Rawalpindi.

Professor of Anaesthesiology
Army Medical College, Rawalpindi.

Advisor in Anaesthesia
Armed Forces of Pakistan.

Dean faculty of Anaesthesiology


College of Physicians and Surgeons of Pakistan.
3

CONTENTS
List of figure vii
Dedication....ix
Forewords....xi
Preface.xiii
Introduction..xv
Lecuter-1
Preoperative Assessment and Premedication....1
Lecuter-2
Inhalational Anaesthetic Agents.........................................................................15
Lecuter-3
Intravenous Anaesthetic Agents ..27
Lecuter-4
Muscle Relaxants ......31
Lecuter-5
Local Anaesthetic Agents ....37
Lecuter-6
Regional Anaesthesia.........................................................................................................................41
Lecuter-7
Fluid Management ........57
Lecuter-8
Acid-Base & Electrolyte Balance ....65
Lecuter-9
Blood Gases, Pulse Oximetry and Capnography.....73
Lecuter-10
Anaesthesia and Related Diseases ...79
Lecuter-11
Cardiopulmonary Resuscitation......85
Lecuter-12
Pain......95
Lecuter-13
ICU...99
Lecuter-14
Complications of Anaesthesia ...103
Lecuter-15
Post Operative Recovery and Care .111
Appendix.....................119
Suggested Reading..131
Index..133

10

11

12

13

LECTURE 1

PREOPERATIVE ASSESSMENT
AND PREMEDICATION
Q:

WHAT IS THE AIM OF PREOPERATIVE ASSESSMENT?

The preoperative management affects outcome for better or worse,


and central to achieving the best possible outcome is a thorough
preoperative evaluation intended to:

Identify the health problems that place the patient at increased risk.
Resolve and control diseases as well as possible.
Define a management plan that minimizes preoperative,
intraoperative, and especially postoperative risks.
The Aim of preoperative evaluation is to reduce morbidity and
mortality.

Q: WHAT POINTS WOULD YOU KEEP IN MIND WHILE TAKING


THE HISTORY?
1. General medical and surgical history. Ask especially about:

2.
3.
4.
5.
6.
7.
8.

Cardiovascular system --- hypertension, angina, orthopnoea, ankle


swelling, previous MI, rheumatic fever, valvular heart disease.
Respiratory system --- shortness of breath, cough, sputum,
wheezing, asthma, tobacco abuse.
Hepatic --- viral hepatitis, jaundice, cirrhosis.
Renal --- renal failure.
Gastrointestinal --- peptic ulcer disease.
CNS --- seizures, peripheral neurological deficit, stroke, muscle
dystrophies.
Musculoskeletal --- osteoarthritis, rheumatoid arthritis.
Endocrine --- diabetes mellitus, thyroid disease.
Haematology --- easy bruising or prolonged bleeding.
Dental --- temporomandibular joint disorder, loose or missing teeth.
Previous anaesthetics and ill effects.
Drug therapy including oral contraceptives and aspirin.
Excessive alcohol intake.
Allergies.
Pregnancy.
Time of last intake of food and drink.
A consent form.

14

Q:

WHAT PHYSICAL EXAMINATION SHOULD BE DONE BY AN


ANAESTHETIST?

1. Respiratory System:
Cyanosis.
Finger clubbing.
Pattern of breathing.
Mediastinal shift.
Localizing signs.
Presence of added sounds on auscultation.
2. Cardiovascular system:
Pulse (rate, rhythm and character).
Venous pressure and character.
Peripheral dependant oedema.
Blood pressure.
Apex beat.
Thrills.
Extra heart sounds and murmurs.
3. State of nutrition, malnutrition and obesity.
4. Skin colour, especially
pigmentation.

pallor,

cyanosis,

jaundice

or

5. Psychological state of the patient, especially anxiety.


6. The airway(The Airway are assessed by Mallampati scoring
system).
7. Ease of venous cannulation.

Q:

WHAT INVESTIGATIONS SHOULD BE AVAILABLE AT


PREOPERATIVE ASSESSMENT?

Depending upon the age and condition of the patient following


investigations may be required:
1.
2.
3.
4.
5.

Urine tests, especially for sugar, ketones and protein.


Haemoglobin and blood count.
Blood urea and Creatinine.
Serum electrolytes.
Blood glucose.
15

6. E.C.G.
7. Chest X-ray.
8. Echocardiogram.
9. Bedside pulse oximetry.
10. Other special investigations may be ordered when indicated.
The above investigations help to assess the status of the patient
condition. The anaesthetist should correct any abnormality in the
investigation before giving anaesthesia. He may refer the patient to
appropriate consultant.

Q: WHAT
IS
ASA
(AMERICAN
SOCIETY
ANAESTHESIOLOGISTS) SCORING SYSTEM?

OF

The ASA scoring system describes the preoperative condition of a


patient. It makes no allowances for the patients age, smoking history,
any obesity or pregnancy. Addition of postscript E indicates emergency
surgery.
THE ASA SCORING SYSTEM
I
Healthy patient.
II
Mild systemic disease, no functional limitations.
III Moderate systemic disease, definite functional limitation.
IV Severe systemic disease that is a continuous threat to life.
V
Moribund patient, unlikely to survive 24 hours with or without
operation.

Q: WHAT IS MALLAMPATI SCORING SYSTEM?


Clinical assessment of airway is very essential. In Mallampati scoring
system, the patient sits opposite the anesthetist with mouth open and
tongue protruded. The structures visible at the back of the mouth are
noted as described below.

Class 1 faucial pillars, soft palate and uvula visible.


Class 2 faucial pillars and soft palate visible, uvula masked by
base of tongue.

Class 3 only soft palate visible.

Class 4 soft palate not visible.


Mallampati scoring system helps the anaesthetist for easy intubation.
Patients in class1 and 2 are intubated easily for other classes
anaesthetist adopted other measures to intubate the patients such as
stylet or fiber optic laryngoscope.

16

Q: WHAT ARE THE EFFECTS OF PRE- EXISTING DURG


THERAPY?
1. Antihypertensive drugs are normally continued up to the time
of surgery, otherwise hypertensive crisis may occur. Adequate
therapy restores a normal blood volume and minimizes the risk
of a dangerous fall of arterial pressure at induction of
anaesthesia. The avoidance of hypovolaemia during surgery is
important. Bradycardia is common in those taking betablocking drugs.
2. Antianginal drugs such as calcium channel blockers or nitrates
should not be stopped before surgery without a very specific
reason, or angina may recur.
3. Lithium should be stopped 2 days before major surgery as it
potentiates the non-depolarizing group of relaxants. In
emergency cases Suxamethonium and regional blocks should
be considered.
4. Monoamine oxidase inhibitors such as phenelzine should be
discontinued 2 weeks before surgery, otherwise hypo or
hypertensive crisis may occur and its prolongs analgesics effect
particularly pethidine and opioids.
5. Levodopa should be continued upto the time of surgery to
prevent the recurrence of severe Parkinsonism, dysphagia and
aspiration pneumonia.
6. Steroid therapy suppresses ACTH production by the anterior
pituitary. In time the adrenal cortex atrophies and is thus
unable to increase its secretions in response to stress. This
results in profound hypotension during and after anaesthesia
with decreased sensitivity to catecholamine. Thus it is generally
safer to assume some diminution of adrenal reserve and to give
extra hydrocortisone over the period of surgery, e.g.
hydrocortisone 100 mg i.m. just before surgery, and continued
6 to 8 hourly for 24 hours after minor surgery, or for 3 days in
case of major surgery.
7. insulin should be continued. The patient should be NPO after
midnight, no IV fluids, and half of the usual morning
subcutaneous dose given. Hypoglycemia and hyperglycemia
should be avoided.
8. Oral contraceptives should be discontinued 4 weeks prior to
surgery as they increase the risk of DVT.

Q: WHAT ARE THE REASONS FOR THE ADMINISTRATION OF


PREMEDICANTS?

To reduce fear and anxiety.

17

To reduce saliva secretion.


To prevent vagal reflexes, due to surgical stimulation or associated
with medication.
For specific therapeutic affects, e.g. steroids, H2 blockers, etc.

Q: WHAT ARE THE


PREMEDICATION?

DIFFERENT

DRUGS

USED

FOR

SEDATIVES
BENZODIAZEPINES
These are all good premedicants and can be given orally producing
sedation, amnesia and freedom from anxiety.
Midazolam: has been used for night sedation before surgery (7.5 15
mg) or as premedication. Dose is 70-100 mcg/kg i.v. 30-60 min before
surgery.
Diazepam: 10-20 mg, orally or i.v. duration 4-8 hrs.
ANALGESICS
Long acting NSAIDs gives useful background analgesia. Ketoprofen
(100-200 mg oral or rectal, 30 mg i.m, i.v.), Piroxicam (20-40 mg oral),
Diclofenac (50-100 mg oral or rectal) will all give useful analgesia in
patients suffering from pain preoperatively. Pethidine or Morphine can
also be used.
ANTICHOLINERGIC AGENTS

ATROPINE

Effects on nervous system:

Competitive blocking action on muscarinic receptors supplied by


postganglionic cholinergic nerves.
Complete vagal blockade requires a dose of 3mg.
Inhibits sweating.
Stimulates the medulla and higher centres.
Effects on eye:
Dilated pupils.
Loss of accommodation.
Effects on respiratory system:
Sweat, bronchial and salivary glands are inhibited.
Bronchodilatation.
Slight increase in anatomical dead space.
Effects on circulatory system.
Tachycardia.
Decreased filling time.

18

Increased myocardial oxygen consumption.


Cardiac output and blood pressure is increased.
Effects on alimentary system.
Tone and peristalsis of gut are decreased.
Increases chances of regurgitation.
Dose: usual adult dose, 0.6 mg i.m. (in children 0.015 mg/kg) 1 hr
before operation. With neostigmine the dose is 1-2 mg.

HYOSCINE HYDROBROMIDE:
Used as a gastrointestinal antispasmodic. It is a tertiary amine, so
crosses the blood-brain barrier and causes sedation. Occasionally it
produces central anticholinergic syndrome. It is a mild respiratory
stimulant, while its actions on iris, salivary, sweat and bronchial glands
are stronger than atropine. It is a moderately powerful antiemetic.
Dose 10-30 mg.

GLYCOPYRONIUM BROMIDE:

It reduces the tone of lower oesophageal sphincter. It suppresses


gastric secretions better than atropine or hyoscine. It causes
tachycardia; so effective in preventing bradycardia due to
suxamethonium. It efficiently dries up salivary secretion.
Dose: premedication 0.2-0.4 mg (adult); 4-8 mcg/kg (child).
Intravenous use to protect against bradycardia (adult) 0.2 mg, 4
mcg/kg (child).
ANTACIDS
These are commonly prescribed for patients thought to be at risk of
regurgitation and aspiration. Ranitidine, 150 mg orally or 50 mg i.m.
DRUGS FOR SPECIFIC EFFECTS
These include all drugs used to ensure optimal treatment of specific
conditions up to the time of surgery, e.g. salbutamol inhalation for
asthmatics.
NOTE: It is usually quoted that sympathetic words and reassurance to
patients act as sedative effect.
Remember:- reassurance + sympathetic words = 10 mg diazepam.

EQUIPMENT
Q1: WRITE SHORT NOTES:-

19

1. Anaesthesia Machine
2. Cylinders
3. Vaporizers
1. ANAESTHESIA MACHINE
Def: Machine which delivers measured amount of gases & volatile
anaesthetic agents from source of supply to patient through tubing.
Basic functions of machine: To deliver compressed gases to patient at a safe pressure.
To allow the flow & composition of the gases to be easily
adjusted.
To permit the addition of a precise concentration of volatile
anaesthetic such as isoflurane.
To deliver this mixture to a common gas outlet & hence, to a
breathing circuit on ventilation.
Types of Anaesthesia Machine: - There are two types of
Anaesthesia Machine.
I)
Continuous Flow: - Machine delivers a mixture of gases
& vapours at a continuous flow set by anaesthetist into a reservoir
bag from which the patient inhales.
II)
Demand Flow: - Machine delivers the preset mixture of
gas at flow rates demanded by breathing pattern of the patient
without interposition of reservoir bag.

20

Fig:

The system is an anaesthetic machine of the Boyles type. Nitrous oxide and oxygen from cylinders on the left
are measured by rotameters (flow meters). Control levers determine what proportion of the total flow goes through the
bottle. A rod raises or lowers the hood.
This is a simple anaesthetic apparatus design by Edmond Boyle. He was commonly known as Cookie. In place of
ether vaporizer (as shown in the figure) these days other vaporizers such as halothane, isoflurane, sevoflurane etc are
installed.

Components of Anaesthesia Machine: Gas inlets receive medical gases from attached cylinders or
hospitals gas delivery system.
Pressure regulators reduce gas pressure.
Oxygen-Pressure failure devices signals low oxygen pressure.
Vaporizers blend gases with volatile anaesthetic agents.

21

A fresh gas out-let delivers the final gas composition to the


breathing circuit.

2. CYLINDERS.

Cylinders are constructed from molybdenum steel.


Cylinders are tested hydraulically every 5 years to ensure that they
can withstand hydraulic pressures considerably in excess of those to
which they are subjected in normal use & the tests recorded by a
mark stamped on the neck of the shoulder. Gas cylinders are tested
by
I) Tensile test.
II) Flattening, impact & band tests.
III) Hydraulic or pressure test.
Filling ratio of a cylinder is the ratio of weight of gas in the cylinders
to weight of water the cylinder could hold. Great care is taken that
the gases are free from water vapours, otherwise when the cylinder
is opened, temperature fall & water vapours would freeze & block
the exit valve.
Cylinder are identified by:i. Size of cylinder e.g. oxygen cylinder are 6 different sizes
C,D,E,F,G,J, & N2O cylinders are 5 different sizes C,D,E,F,G
ii. Colour Codes
a.
N2O cylinder has Blue body & Shoulder.
b.
O2 cylinder carries black body & white shoulder.
c.
CO2 cylinder has grey body & shoulder.
iii.Pin Index System is a device to prevent interchangeability of
cylinders of different gases. The pegs on the inlet connection slot
into corresponding holes (pits) on the cylinder valve.
e.g. position of pit on cylinders:
O2
--2, 5
N2O --3, 5
CO2 --1, 6
Different gas cylinders carry different pressures e.g. O 2 cylinder
pressure is 137 bars N2O cylinder pressure is 44 bar at 15C.
Cylinder valves should be opened slowly to prevent sudden
surges of pressure & should be closed with no more force that is
necessary otherwise valve seating may be damaged.

3. VAPORIZERS
Definition: - A vaporizer is a device for adding clinically useful
concentration of anaesthetic vapours to a stream of carrier gas.
Types:-

22

i.

Drawover vaporizers:
In this type of
vaporizers, gas is pulled through the
vaporizer
when
the patient inspires, creating a
subatmospheric pressure.
Resistance to gas flow through a draw over vaporizer
must be extremely small.
ii.
Plenum Vaporizers: - In this type of vaporizers
gas is forced through the vaporizer by the pressure of fresh
gas supply.
Resistance of plenum vaporizers may be high enough to
prevent its use as draw over vaporizers.
Principles of both devices are similar. All the anaesthetic gas
entering the vaporizer passes through the anaesthetic
liquid and becomes saturated with vapour. 1 ml of liquid
anaesthetic is equivalent of approximately 200ml of
anaesthetic vapors.
Concentration of anaesthetic in the gas mixture emerging
from the outlet port is dependent upon:

Saturated Vapour Pressure of the anaesthetic


liquid in the vaporizer.
Temperature of liquid anaesthetic agent, as this
determines its saturated vapour pressure.
Splitting ratio i.e. the flow rate of gas through
the vaporizer chamber in comparison with that through bypass.
Surface area of anaesthetic agent in the
vaporizer.
Duration of use as the liquid in the vaporizing
chamber evaporate, its temperature falls & thus its saturated
vapour pressure decreases. This leads to reduction in
concentration of anaesthetic in mixture leaving the exit port.
Nature of liquid
Fresh gas flow

Q2: WRITE SHORT NOTE ON ENDOTRACHEAL TUBES?


ENDOTRACHEAL TUBES
There are many designs of endotracheal tubes. The general
considerations determining their construction as follow.
MATERIAL

Red rubber
-

Not normally disposable

23

- Relatively irritant, and not ideal for prolonged


Intubation
- Firm/curvature predetermined
- May transmit infection
Plastic (PVC)
- Disposable
- Non-irritant (implantation-tested)
- Moulds to body contours at 37C
CUFFS
Red rubber cuffs are firm and rounded so that a seal between the
endotracheal tube and the trachea exists over small areas. The
mucosa is likely to be damaged, not only because of the
chemical irritants but also because of compression, and hence
hypoxia, of the mucosa.
PVC tubes have cuffs of varying shapes. The shape of the
cuff can be more cylindrical, thus, by increasing the area of seal,
there is a reduction in the pressure necessary in the cuff.
The seal between tracheal mucosa and endotracheal tube
is required to prevent the escape of gas (during IIPV) and also to
prevent the aspiration of saliva or gastric contents into the
tracheobronchial tree.
Age/4 + 4.5 is the accepted formula for determining the size
(mm) of the endotracheal tube, for a child.
The length of the tube for a child is determined by:
Age/2 + 12 cm (oral)
Age/2 + 15 cm (nasal)

Q3: WRITE A SHORT NOTE ON LARYNGOSCOPES


LARYNGOSCOPES
These are the instruments to see larynx.

There are many designs for use, depending on requirement:

1.

NEONATAL STRAIGHT BLADE


The epiglottis is relatively large and floppy; a straight blade is
necessary to flatten and hold the epiglottis forward to allow
the cords to be visualised.
2.
INFANTSTRAIGHT OR CURVED BLADE
The tongue of the infant is large in relation to the buccal
cavity and blade design is aimed at keeping it out of the way.
Blades which are almost tubular are used in infants with tissue
flaps associated with palatal defects. The most commonly
used paediatric laryngoscopes are the Anderson, Magill and
the Robertshaw.

24

25

Fig: INTRODUCING A LARYNGOSCOPE.

26

Fig: HOW TO USE A LARYNGOSCOPE. (A) Insert the laryngoscope with your wrist straight, then extend
your wrist. (B) Finally, lift the patients jaw forwards. (C)The secret of success is to have the patients head
extended on his neck before you begin. (D) and to have his neck flexed forwards. (E). Arrange the pillow
under his neck and shoulders so that you can achieve this. This has been likened to the position of sniffing
the morning air.

3.

ADULT STRAIGHT OR CURVED BLADE


The primary aim is deflection of the tongue from the line of
vision of the vocal cords; however, a variety of other problems
have been overcome.
a. A laryngoscope with an obtuse angle between the handle
and the blade to facilitate insertion into the mouth of
patients with difficult access, e.g. in an iron lung, in severe
fixed flexion or in a halo splint for stabilization of the
cervical spine.
b. A left handed blade for use is patients where the right
side of the mouth is invaded by tumour, or access is
otherwise compromised.
c. The addition of a prism to the blade allows the vocal cords
to be visualized when they are not in direct line of sight.
d. McCoy
Laryngoscope.
Resembles
a
conventional
laryngoscope, but the distal part of the blade is hinged and
can be tilted up or down by a lever on the handle. Allows
27

the larynx to be lifted to improve vision in case of


difficulty.
4. LARYNGEAL MASK AIRWAY (LMA)
A revolution in airway control. The LMA is inserted into the
mouth and advanced until it comes to lie against the posterior
pharyngeal wall opposite the larynx. The large cuff is then
inflated and this creates a seal around the laryngeal opening.
The seal of airway to trachea is not so reliable as when using
an endotracheal tube and a number of studies have shown
some leakage past the LMA which could potentially enter the
trachea. Some doubts have been expressed as to the
suitability of the LMA for use during controlled ventilation and
for surgery within the mouth and pharynx, e.g., tonsillectomy.
Nevertheless, it has been used widely for these situations.
Great care must be taken to ensure that airway inflation
pressures remain low if using an LMA for controlled
ventilation.
5. FIBREOPTIC LARYNGOSCOPE
A thin flexible fibreoptic device that will pass through a
tracheal tube. The fiberscope is passed through the nose or
mouth (the nose is usually easier) and advanced under direct
vision until it lies within the trachea. The tracheal tube, which
has been previously slid onto the fiberscope is then advanced
using the fiberscope as a guide. The fiberscope is then
withdrawn. The use of the fibreoptic laryngoscope requires
previous training. It is the safest technique for securing the
airway is case of anticipated difficult Intubation and may be
performed with the patient awake following local anaesthesia
to the airway.

28

Fig:

THE POSITIONS OF PTIENTS OF DIFFERENT AGES DURING INTUBATION. Put the pillow under an
adults head and neck, but under a childs back.

29

LECTURE 2

INHALATIONAL ANAESTHETIC AGENTS


Q: WHAT ARE THE CHARACTERISTICS OF AN IDEAL VOLATILE
ANAESTHETIC AGENT?
The characteristics of an ideal volatile anaesthetic agent are:
1. Non-inflammable, non explosive.
2. Stable physical characteristics.
3. Appropriate volatility, having low boiling point and high SVP.
4. Potent.
5. Cardiovascular and respiratory stability.
6. Low blood-gas solubility giving rapid induction/recovery.
7. Analgesic.
8. Low incidence of nausea and vomiting.
9. Should not sensitize the myocardium to adrenaline.
10.
Non-irritant and pleasant smelling.
11.
Non-metabolized.
12.
Non-hepatotoxic and nephrotoxic.
13.
Non-teratogenic.
14.
Cheap.
15.
No effect on CBF and intracranial pressure or effect easily
antagonized by hyperventilation.
16.
No effect on E.C.G.

30

Fig: HOLDING THE MASK WITH ONE HAND.

Fig: HOW TO HOLD THE MASK WITH TWO HANDS.

31

Q: WHAT IS THE CLASSIFICATION


ANAESTHETIC AGENTS?

OF

INHALATIONAL

A: VOLATILE ANAESTHETIC AGENTS


1. ETHERS.
Methoxyflurane (not in use due to its nephrotoxic effects).
Enflurane.
Isoflurane.
2. HALOGENATED HYDROCARBONS.
Halothane.
Sevoflurane.
Desflurane.
Chloroform(not used these days due to its toxic effects).

B: GASEOUS ANAESTHETIC AGENTS

1. NITROUS OXIDE .
2. CYCLOPROPANE (not used these days due to its toxic effects and
explosion hazards).

NITROUS OXIDE
Q: HOW IS NITROUS OXIDE PREPARED?
Nitrous oxide is also known as laughing gas. It is prepared
commercially by heating Ammonium Nitrate crystals to a temperature
of 245-270oC.
Heat
NH4NO3 ------N2O + H2O

Q: WHAT ARE THE IMPURITIES FORMED ALONG WITH N2O?


The chief impurities are:
Nitrogen which dilutes the anaesthetic.

Nitric oxide which combines with hemoglobin to produce anemic


type of hypoxia.

Nitric acid causes pulmonary oedema.

Nitrogen dioxide which may damage the valves.

Ammonia.

Nitrous acid.
Now-a-days 99.5% pure gas is supplied.

32

Q: HOW IS NITROUS OXIDE STORED?


Nitrous oxide is stored in compressed form as a liquid in blue cylinders
at a pressure of 50 bar (5000kPa; 750 lb.in 2). Because the cylinder
contains liquid and vapour, the total quantity of nitrous oxide
contained in cylinder can be ascertained only by weighing. Nitrous
oxide cylinders should be kept in a vertical position during use so that
the liquid phase remains at the bottom of the cylinder.

Q: WHAT ARE THE PHYSICAL PROPERTIES OF NITROUS


OXIDE?

Sweet smelling.
Non irritating.
Colorless.
Non inflammable but supports combustion.
Formula
N2 O
Molecular wt:
44
Boiling point:
-89oC
Critical temperature:
36.5oC
Critical pressure:
71.7 Atm.
Blood/gas solubility coefficient:
0.468
Eliminated unchanged from the body mostly via lungs.
Stable.
Not affected by soda lime.

Q: WHAT ARE THE EFFECTS OF NITROUS OXIDE ON VARIOUS


SYSTEMS OF THE BODY?
1. CNS
Causes CNS depression.
Paralysis of respiratory and vasoactive centre does not occur.
2. RESPIRATORY SYSTEM
Respiration is stimulated (both depth and rate).
Reduces the MAC of volatile anaesthetics by about 50%.
3. MUSCULAR SYSTEM

Depression of skeletal system is minimal.

4. MISCELLANEOUS
No effect on kidney or liver function.
Nausea and vomiting are likely to occur.
33

Crosses placental barrier


depression in fetus.

but

does

not

cause

respiratory

Q: WHAT ARE THE SIDE EFFECTS OF NITROUS OXIDE?

May cause exhilaration and euphoria during induction.


Unpleasant hallucinations and dysphoria can occur.
Continual use for days or weeks may cause neutropenia or
macrocytic anaemia.
Interferes with nucleic acid synthesis.
Diffuses into cavities and cause otological disturbances in middle
ear.
Prolonged use can lead to bone marrow depression and
teratogenic effect.
Respiratory depression and diffusion hypoxia.
Pollution problem.
Expensive.
Difficulties of cylinder transport.

HALOTHANE
Q: WRITE A SHORT NOTE ABOUT THE CHEMISTRY AND
PHYSICAL
PROPERTIES OF HALOTHANE?
Halothane is 2-bromo-2-chloro-1, 1, 1-triflouroethane. Its formula is
CI
F

H------- C ----- C -------F

Br
F
The

physical properties are:Halogenated hydrocarbon.


Non inflammable and non explosive.
Colourless and sweet smelling.
Unstable in light.
Stored in amber coloured bottles with thymol as preservative.
Mol. wt:
197
o
Boiling point:
50.2 C
L.H.V:
35.2 Calories/gm
Oil water solubility: 220
34

Blood gas solubility:


2.5 at 37oC
MAC:
0.75 V %

Q: WRITE DOWN BRIEFLY THE EFFECTS OF HALOTHANE ON


VARIOUS ORGAN SYSTEMS?
CVS:

Blocks sympathetic ganglion


Increases vagal tone causing bradycardia
Direct myocardial depressant effect.
Direct depressant of vasomotor center.
Increases impulse discharge from baroreceptors.
Depresses S-A node.
Direct depressant of vasculature of smooth muscles.
Dose dependent hypotension due to decreased cardiac output and
lowered peripheral resistance.
Sensitizes heart to arrhythmic effects of adrenaline.
Coronary artery vasodilator.

CNS
Increases the CSF pressure and cerebral blood flow.
Blunts autoregulation of cerebral blood pressure.
Not a very good analgesic.
RESPIRATORY SYSTEM
Depresses respiration with shallow rapid breathing.
Rate increases with depth of anaesthesia.
Bronchodilator.
Increases apneic threshold.
Hypoxic drive depressed.
Attenuates airway reflex.
Depresses clearance of mucous secretions from respiratory tract.
MUSCULAR SYSTEM
Potentiates the effect of non depolarizing muscle relaxant.
Moderate relaxation.
Triggering agent for malignant hyperpyrexia.
UTERUS

Uterine relaxation and bleeding only in parturient uterus.

35

LIVER

Halothane hepatitis.
Decreases hepatic blood flow.
Slows down the metabolism of drugs like fentanyl, phenytoin,
verapamil.

HORMONAL EFFECTS

Increase in growth hormone, serum thyroxine.


Sensitivity to insulin is increased.

BODY TEMPERATURE

Causes 1o C drop of esophageal temperature and 4o C rise of skin


temperature.

MISCELLANEOUS
HALOTHANE SHAKES: recovery from halothane is sometimes
associated with restlessness or shivering. Cover with blankets and
ensure adequate oxygenation.

Q: WHAT ARE THE CONTRAINDICATIONS TO THE USE OF


HALOTHANE?

Patient with hepatic dysfunction.


Patient with increased intracranial pressure.
Patient having history of malignant hyperpyrexia.
Patients with hypovolaemia and severe cardiac disease such as
aortic stenosis.

ENFLURANE
Q: WRITE A SHORT NOTE ABOUT THE CHEMISTRY AND
PHYSICAL
PROPERTIES OF ENFLURANE.
Enflurane is defluoro methyl ether of 1, 1, 2 trifluro-2-Chloroethane.
F

H ----- C ---- O ----- C ----- C ------ H

F
E
CI
The physical properties of enflurane are:-

36

Stable, colourless without added chemical stabilizers.


Non inflammable, non explosive.
Pleasant ethereal smell.
Does not decompose when circulated with oxygen and water
vapours through warm soda lime.
Blood/gas solubility coefficient: 1.8
MAC:
1.7%
Boiling point:
56oC
SVP:
175 mmHg at 20oC

Q: WHAT ARE THE EFFECTS OF ENFLURANE ON VARIOUS


ORGAN SYSTEMS?
CVS

Dose dependant depression of myocardial contractility.


Reduction in cardiac output.
Less likely to sensitise the heart to adrenaline.
Dose dependant reduction in arterial pressure.
No central vagal effect.
Hypotension leads to reflex tachycardia.
Preferable
to
halothane
during
surgery
involving
pheochromocytomas and other tumours associated with excessive
secretion of catecholamines.

RESPIRATORY SYSTEM

Non irritant.
Does not increase salivary or bronchial secretions.
Dose dependant depression of alveolar ventilation with reduction
in tidal volume and an increase in ventilatory rate.
Pharyngeal and laryngeal reflexes are diminished quickly.

UTERUS
Dose related relaxation of uterine muscle.
CNS

Dose dependant depression of EEG activity.


Produces epileptiform spike activity.
Twitching of face and arm muscles.
Avoided in epileptic patients.

MUSCLE RELAXATION

37

Enhances the effect of non-depolarizing muscle relaxants.

Q: WHAT ARE
ENFLURANE?

THE

INDICATIONS

FOR

THE

USE

OF

For induction and maintenance of general anaesthesia.


For dental anaesthesia in view of rapidity of action and recovery
with stability of cardiovascular system.
May be used in children.

38

ISOFLURANE
Q: WHAT IS THE FORMULA AND THE MAJOR PHYSICAL
CHARACTERISTICS OF ISOFLURANE?
Isoflurane, which is 1-chloro-2, 2, 2-triflouroethyl diflouromethyl ether,
is an isomer of enflurane. Its formula is
F

H ----- C ------ C ----- O ----- C ------ H

F
CI
F
Its physical properties include
Colorless, volatile anaesthetic
Slightly pungent odour
Does not require preservatives
Non inflammable
Vapor pressure
MAC
1.2
Blood/gas partition coefficient 1.4

240 mmHg at 20oC

Q: WHAT ARE THE EFFECTS OF ISOFLURANE ON VARIOUS


ORGAN SYSTEMS?
RESPIRATORY SYSTEM
Dose dependant depression of ventilation.
Decrease in tidal volume with increase in ventilatory rate.
Blunts response to hypoxia and hypercapnia.
Bronchodilator.
CVS

Myocardial depressant but less depression of cardiac output than


halothane potent peripheral vascular dilator.
Systemic hypotension due to reduction in systemic vascular
resistance coronary vasodilatation leading to Coronary Steal
Syndrome.
Does not sensitize the myocardium to catecholamines.

CNS

39

High inspired concentrations, Minimum Alveolar Concentration


(MAC) > 1 lead to vasodilatation, an increased cerebral blood flow
and intracranial pressure.
No seizure activity on EEG.
Does not blunt autoregulation.
Decreases CMRO2 (Cerebral Metabolic Rate of Oxygen
consumption).

MUSCULAR SYSTEM

Dose dependant depression of neuromuscular transmission with


potentiation of non depolarizing neuromuscular blocking drugs.

RENAL

Decreases renal blood flow, GFR and urine output.

HEPATIC

Total hepatic blood flow is decreased but to lesser extent than


halothane
LFTs minimally affected.

Q: WRITE SHORT NOTE ON SEVOFLURANE.


SEVOFLURANE
This is non-flammable ether. It is
devoid of significant
cardio/respiratory side-effects. The major advantage is that its very low
blood: gas solubility coefficient (0.6) allows its use for rapid face mask
induction of anaesthesia, especially in children. It is in wide clinical use
in Japan.
It is 3% metabolised.

MAC
Blood: gas solubility at 37C
Boiling point
Saturated vapour pressure at 20C
mmHg

2.0
0.65
58.5C
170

Q: WRITE SHORT NOTE ON DESFLURANE


DESFLURANE
Desflurane is also halogenated ether and is licensed for use in Europe
and North America. It is not unpleasant to inhale and is non-irritant to
the respiratory tract at low concentrations. It has a very low blood; gas

40

solubility (0.42) and is thus associated with short induction and wakeup times. It is 0.02% metabolised.
MAC
6.0
Blood: gas solubility at 37C
0.45
Boiling point
22.8C
Saturated vapour pressure at 20C
66
mmHg
Both these agents are expensive to produce. They offer advantages
over other anaesthetic vapours but sevoflurane produces a toxic
product on contact with soda-lime whilst desflurane increases heart
rate and is a respiratory irritant at concentrations > 1MAC.

41

LECTURE 3

INTRAVENOUS ANAESTHETIC AGENTS


Q: DEFINE INTRAVENOUS ANAESTHETIC AGENTS?
I/V anaesthetic agents may be defined as drugs that will induce loss
of consciousness in one arm brain circulation time when given in
appropriate dosage.

Q: NAME THE COMMON INTRAVENOUS AGENTS.


Common I/V anaesthetic agents include:
1.
2.
3.
4.
5.

Thiopentone sodium
Ketamine.
Propofol.
Etomidate.
Methohexitone.

Q: WRITE A SHORT NOTE ON THIOPENTONE?


Thiopentone sodium:
It is the most commonly used I/V anaesthetic agent. It is usually
used for induction of anaesthesia. It is a sodium salt of barbituric acid
and is the sulphur analogue of pentobarbitone. The 2.5% solution,
which is commonly prepared, has a pH of 10.5.

It produces anaesthesia usually in less than 30 sec. after i/v


injection.
Duration of action is 5 10 min.
Myocardial contractility is depressed and peripheral vasodilatation
occurs, which leads to Hypotension.
Ventilatory drive is decreased and a short period of apnoea is
common, preceded by a few deep breaths.
Skeletal muscle tone is reduced due to suppression of spinal cord
reflexes.
Intraocular Pressure (IOP) is reduced by 40%
Antanalgesic
If injected accidentally in the artery it will cause sever pain in the
fingers. One should keep the needle in-situ and inject papaverine
(Vasodilator) and local anaesthetic procaine to relief pain.

INDICATIONS:

42

Induction of anaesthesia, Maintenance of anaesthesia, Basal


narcosis by rectal administration, Status epilepticus, Reduction of
intracranial pressure.
ABSOLUTE CONTRAINDICATIONS:

Airway obstruction.

Porphyria

Previous hypersensitivity reaction.


DOSE:

3 5 mg/kg body weight as 2.5% solution.

Q: WRITE A SHORT NOTE ON KETAMINE?


KETAMINE:
This is a phencyclidine derivative. It produces dissociative
anaesthesia.

After i.v. injection it induces anaesthesia in 30 60 sec. duration of


action is 10 15 min. After i.m. injection the effect starts within 3-4
min. and duration of action is 15 25 min.

There may be emergence delirium, restlessness, disorientation,


nightmares and hallucinations.

Arterial pressure increases by up to 25%.

Heart rate increases by upto 20%. Myocardial oxygen demand also


increases.

Transient apnoea may occur after i.v. injection, but ventilation is


well maintained thereafter.

Pharyngeal and laryngeal reflexes and a patent airway are well


maintained.

Muscle tone is increased & spontaneous movements may occur.

IOP increases.
Dosage:

2-mg/kg i.v. for induction.


1 1.5 mg/kg for maintenance.
8 10-mg/kg i.m.

PROPOFOL

Q: WHAT IS THE FORMULA OF PORPOFOL?

43

2, 6, di-isopropylphenol: 1% solution in egg white lecithin emulsion.

Q: WHAT
ARE
PRESENTATION?

THE

PHYSICAL

PROPERTIES

AND

Propofol is extremely lipid soluble, but almost insoluble in water. It is


formulated in a white, aqueous emulsion containing soybean oil and
purified egg phosphatide.

Q: WHAT ARE THE PHARMACOKINETICS OF PROPOFOL?

Distributed rapidly
Termination of action occurs by redistribution
Metabolized at both hepatic and extra hepatic sites
Very high clearance
Excretion through kidneys

Q: WHAT IS THE DOSE OF PROPOFOL?

2-3 mg/kg IV induction


100-200 g/kg/min maintenance
Sedation: 25-100 g/kg/min

Q: WHAT ARE
PROPOFOL?

THE

PHARMACOLOGIC

ACTIONS

OF

CNS
EEG frequency decreases and amplitude increases
Cerebral blood flow, intracranial pressure and cerebral metabolic oxygen
demand decreases
May have anticonvulsant effect
Occasional excitatory activity
CVS
Venous dilatation, decreased
depression lead to hypotension
Heart rate may increase

peripheral

resistance

and

cardiac

RESPIRATORY
Transient apnoea
44

Decreased rate and tidal volume


Depression of laryngeal reflexes more than barbiturate
HEPATIC
None
RENAL
Decreased cardiac output may decrease renal blood flow
MISCELLANEOUS
Less postoperative nausea than barbiturates
Possible antipruritic effect at low dose

Q: WHAT ARE THE ADVERSE EFFECTS?


1.
2.
3.
4.
5.
6.
7.

Very low incidence of anaphylaxis


Caution if lipid disorder present
Cardiovascular depression
Respiratory depression
Excitatory phenomena
Pain on injection
Occasional monoclinic movements.

45

LECTURE 4

MUSCLE RELAXANTS
Q:

CLASSIFY MUSCLE RELAXANTS.

Muscle relaxants are classified as:


1. Depolarizing Muscle Relaxants

Succinyl choline (commonly used)


Decamethonium

2. Non-Depolarizing Muscle Relaxants.

Long Acting

Intermediate Acting

Tubocurarine
Doxacurium
Pancuronium (commonly used)
Gallamine (not used due to its ganglion blocking effects)

Atracurium (commonly used)


Vecuronium
Rocuronium

Short Acting
Mivacurium

Q: WHAT ARE
DEPOLARIZING
1.
2.
3.
4.
5.
6.
7.
8.

THE SALIENT FEATURES


MUSCLE BLOCK?

OF

NON-

Do not cause muscle fasciculation.


Very hydrophilic.
Relatively slow onset.
Reversed by neostigmine and other anticholinesterases.
Effects reduced by acetylcholine and adrenaline.
Potentiated by volatile agents, Mg2, and hypokalemia.
Mild cooling antagonizes their effects.
Acidosis increases duration and degree of non-depolarizing block.

46

Q: WHAT ARE THE SALIENT FEATURES OF DEPOLARIZING


MUSCLE BLOCKING DRUGS?
1.
2.
3.
4.
5.
6.

Cause muscle fasciculations but not in myasthenia gravis.


Repolarization is interfered with; the resting membrane potential
is held up until phase II block develops, when it returns to -70
mV.
Not reversed with neostigmine and other anticholinesterases.
potentiated with isoflurane, enflurane, Ach, respiratory alkalosis,
hypothermia and Mg2+.
Antagonized by ether, halothane, acidosis and non-depolarizing
relaxants.
Fast dissociation constants at receptor.

Q: WHAT CLINICAL SIGNS INDICATE THE NEED OF MORE


RELAXANT
DURING SURGERY?
1.
2.
3.
4.
5.

Hiccup, due to contraction of periphery of the diaphragm.


Rigidity of abdominal wall.
Increased resistance to inflation of lung.
Bucking or coughing on tracheal tube.
As indicated by neuromuscular monitoring.

Q: WHAT ARE THE CLINICAL SIGNS OF INCOMPLETE


REVERSAL?
1.
2.
3.

6.
7.

Shallow respiration.
Jerky respiration.
Tracheal tug and see-saw respiration where, as the abdomen
moves out, the chest moves in.
Cyanosis.
A restless, frightened, struggling patient, who says that he or she
cannot breathe.
Diplopia.
Inability to raise head or extrude tongue.

Q:

WRITE A SHORT NOTE ON ATRACURIUM?

4.
5.

ATRACURIUM
Physical structure:
It is an isoquinolon compound belonging to quaternary group.
Pharmacokinetics:
Absorption: from I/M and I/V routes.
47

Distribution:
Throughout ECF.
Metabolism:
Hoffmann degradation.
Alkaline ester hydrolysis in plasma.
Pharmacodynamics:
Dose: 0.5-mg/kg i.v. as bolus dose.
Top ups 0.3- 0.1mg/kg i.v.
Neonates are slightly more resistant so dose is 0.3 mg/kg.
Speed of onset: 1-2 min.
Duration: 20-40 min
.
Reversed with: neostigmine.
Side effects and clinical considerations
1. Release of histamine.
2. Hypotension and tachycardia, if given in excess of 0.5mg/kg
bronchospasm so avoid in patients with bronchial asthma.
3. Laudanosine, a breakdown product of Hoffmann degradation, is
epileptogenic.
4. Duration of action can be markedly prolonged in hypothermia and
acidotic patients.
5. Atracurium precipitates as a free acid if given into an i.v. line
containing an alkaline solution such as thiopentone.

Q:

WRITE A SHORT NOTE ON PANCURONIUM?

Physical structure:
It is a long acting quaternary amino-steroid, devoid of hormonal
activity. It resembles two acetylcholine molecules bound together.
Pharmacokinetics:

Strongly bound to gamma globulin and moderately bound to


albumin.
Metabolized by deacytylation in liver to limited degree.
Excretion primarily through kidney (40%) but 10% is cleared
through bile.
Patients with renal failure show prolonged block.
Patients with cirrhosis require increased loading dose due to large
volume of distribution but decreased maintenance dose due to
decreased plasma clearance.

Dose:
0.05mg/kg i.v. bolus. Duration: 40-60 min.
Side effects and clinical considerations

48

1.
2.
3.
4.
5.

Can cause stimulation of the myocardium with rise in pulse rate


and blood pressure.
Vagal blockade and catecholamines release.
Increased incidence of ventricular dysrythmias.
Releases histamine from tissues.
Should be avoided in renal failure and total biliary obstruction.

Q: WHAT IS SUXAMETHONIUM
PHARMACOKINETICS?

AND

WHAT

IS

ITS

Suxamethonium is a dicholine ester of succinic acid. It belongs to


quaternary ammonium group.
Pharmacokinetics:
Absorption: I/V, I/M or S/C.
Distribution: throughout the ECF and slightly through
placenta.
Metabolism: hydrolysis by plasma cholinesterases.
Dose:
1-1.5 mg/kg
Duration: 10-15 min.

the

Q: WHAT ABNORMALITIES CAN OCCUR IN SUXAMETHONIUM


METABOLISM, WHICH CAN PROLONG ITS DURATION OF
ACTION?
1.

2.

Abnormal plasma cholinesterase (inherited)


Atypical cholinesterase.
Fluoride resistant cholinesterase.
Silent gene.
Plasma cholinesterases deficiency:
Acquired
After X-ray therapy
After organophosphorous poisoning
In hyperpyrexia
In cardiac failure
Uraemia
Hypoproteinemia
Myasthenia gravis
Pregnancy
Myxoedema
Asthma
Obesity
Following treatment with: cyclophosphamide, ecothiopate,
ketamine, pancuronium, MAO inhibitors, and oral
contraceptives.

49


3.

Congenital

Plasma cholinesterase Antagonism: by anticholinesterases


such as neostigmine.

Q: WHAT ARE THE SIDE EFFECTS OF SUXAMETHONIUM.


1. PROLONGED APNOEA:
The

commonest causes are:


Atypical serum cholinesterases.
Dehydration and electrolyte imbalance.
An overdose of muscle relaxant.
A low serum cholinesterase level in blood.
An excessive formation of succinyl monocholine.
Dual block.

The management includes

2.

Artificial ventilation and sedation are maintained until monitoring


shows that the block has worn off.
A blood sample is taken for cholinesterase analysis.
Fresh frozen plasma given.
HYPERKALEMIA

A rise in serum potassium of 0.2-0.4 mmol/l occurs due to release


from muscle, especially in burn patients.
3.

RAISED INTRA-OCCULAR PRESSURE


Suxamethonium, 1 mg/kg, raises the pressure an average of 7 mm

Hg.
4.

MUSCLE PAIN
More frequent in women and middle-aged patients.

5.

RAISED INTRAGASTRIC PRESSURE

6.

MALIGNANT HYPERPYREXIA
Incidence is 1 in 100000 adults.

7.

EXACERBATES DYSTROPHIA MYOTONIA

8.

DIRECT
MYOCARDIAL
DEPRESSANT
BRADYCARDIA AND CARDIAC ARREST
MUSCARINIC EFFECTS

9.

LEADING

TO

10. ANAPHYLAXIS

50

Q: WHAT
ARE
THE
SUXAMETHONIUM?

INDICATIONS

FOR

Endotracheal intubation.
ECT.
Short orthopaedic procedures.
Short surgical procedures.

Hyperkalemia.
Known case of atypical pseudocholinesterase.
Hypersensitivity.
In patients with increased intraocular pressure.
Family history of malignant hyperpyrexia.

USE

OF

Q: WHAT ARE THE CONTRA-INDICATIONS TO THE USE OF


SUXAMETHONIUM?

51

LECTURE 5

LOCAL ANAESTHETIC AGENTS


Q: WHAT ARE LOCAL ANAESTHETIC AGENTS?
Local anaesthetics cause reversible blockade of peripheral nerve
conduction or inhibition of excitation at nerve endings with resultant
loss of sensation in the particular area of the body.

Q: HOW DO YOU CLASSIFY LOCAL ANAESTHETIC AGENTS.


A)

ACCORDING TO STRUCTURE
1. HAVING ESTER LINKAGE

Chloroprocaine

Cocaine

Procaine

Tetracaine
2. HAVING AMIDE LINKAGE
Lignocaine
Bupivacaine
Etidocaine
Cinchocaine

B)

ACCORDING TO POTENCY
1. LOW POTENCY AND SHORT DURATION
Procaine
Chloroprocaine
2. INTERMEDIATE POTENCY AND DURATION

Mepivacaine
Prilocaine
Lignocaine

3. HIGH POTENCY AND LONG DURATION

Tetracaine
Bupivacaine
Etidocaine

52

Q: WHAT IS THE SITE OF ACTION OF LOCAL ANAESTHETIC


AGENTS?
The site of action of local anesthetics drugs is at the surface
membrane of cells of excitable tissues. In a myelinated nerve the site
of action is the node of Ranvier.

Q: WHAT IS DIFFERENTIAL BLOCK?


The minimum concentration of local anaesthetic drug necessary to
cause block of a nerve fiber of given diameter is known as the Cm. The
thicker the diameter of a nerve fibre the greatest the Cm required. In
practice the sequence of blockade is autonomic, sensory, and finally
motor block according to fibre diameter.

Q: WHAT DOES THE UPTAKE OF LOCAL ANAESTHETICS


DEPEND UPON?
Local anaesthetic drugs are lipid-soluble bases, which act by
penetrating lipo-protein cell membranes in the non-ionized state. The
blocking quality of a local anaesthetic drug depends on its:

Potency.
Latency (time between its injection and maximum effect) this in
turn depends on nerve diameter, local pH, diffusion rate and
concentration of local drug.
Duration of action.
Regression time (time between commencement and completion
of pain appreciation).

Q: WHAT
TOXICITY?

FACTORS

INFLUENCE

LOCAL

ANAESTHETIC

Quantity of solution.
Concentration of drug.
Presence or absence of adrenaline.
Vascularity of site of injection.
Rate of absorption of drug.
Rate of metabolism of drug.
Hypersensitivity of patient.
Age, physical status and weight of patient.

53

Q: WHAT ARE THE SIGNS OF TOXICITY IN VARIOUS ORGAN


SYSTEMS?
CENTRAL NERVOUS SYSTEM

Central stimulation followed by depression.


Restlessness.
Hysterical behaviour.
Vertigo.
Tremors.
Convulsions.
Respiratory failure.

Treatment: Artificial ventilation with O2 or air.


Intravenous injection of Suxamethonium or
Thiopentone to control convulsions (10-150 mg).
Diazepam.

just

sufficient

CARDIOVASCULAR SYSTEM

Hypotension.
Acute collapse primary cardiac failure, feeble pulse and
cardiovascular collapse, bradycardia, pallor, sweating and
hypotension.
Treatment: Elevate legs.
Give oxygen by IPPV.
Rapid intravenous infusion.
Raise blood pressure.
Cardiac massage.
RESPIRATORY SYSTEM

Apnoea.
Medullary depression.
Respiratory muscle paralysis.

ALLERGIC PHENOMENA

Bronchospasm.
Urticaria.
Angioneurotic oedema.
Cross sensitivity.

54

Q: HOW CAN YOU IMPORVE DURATION AND QUALITY OF


LOCAL
ANAESTHETIC?

Addition of adrenaline, 1:200 000 to 1:500 000 solution.


Injection closer to nerve.
The amount of free base.
Adjusting the pH to about 7.

Q: WHAT ARE
ANESTHESIA?

THE

DIFFERENT

METHODS

OF

LOCAL

Simple topical application at operative site.


Infiltration analgesia to abolish pain.
Field block.
Nerve block (conduction anaesthesia).
Refrigeration analgesia (Cryoanalgesia).
Intravenous local analgesia.
Central neural blockade.

Q: WHAT IS THE MAXIMUM DOSE OF LOCAL ANAESTHETICS?


Drug
Lignocaine
Bupivacaine

Maximum Dose mg/kg


3 to 7 (with adrenaline)
2.5

Remember: 1 ml of 1% lignocaine = 10 mg

55

Lecture 6
REGIONAL ANAESTHESIA
Q:

What is the anatomy of spinal cord?

Spinal cord is a part of CNS, extending from foramen magnum to lower


border of L1 or upper border of L2 in adults and L3 L4 in children. It is
covered by fibro fatty tissue known as meninges. It consists of grey
mater and white mater which represent ascending and descending
tracts. There are 8 cervical. 12 thoracic, 5 lumbar and 5 sacral spinal
nerves.

SPINAL ANAESTHESIA
Q:

DEFINE SPINAL ANAESTHESIA?

Spinal anaesthesia is a type of regional block in which local anaesthetic


is injected into the subarachnoid space. It causes major conduction
block, which refers to blockade of spinal nerve roots. The resultant
nerve block provides surgical anaesthesia as far cephalad as upper
abdomen.

Q:

WHAT ARE THE INDICATIONS OF SPINAL ANAESTHESIA?

Indications are:i)
ii)
iii)

Q:

Lower abdominal surgery e.g. Cesarean Section, Herniotomy,


Transvesical prostatectomy, TURP, Cystoscopies etc.
Perineal surgery e.g. haemorrhoidectomy, fistulectomy, TURP,
cystoscopies etc.
Lower limb surgery e.g. Arthroscopy, amputation, open
reduction internal fixation (ORIF) of fractures etc.

AT WHICH LEVELS THE BLOCK IS PERFORMED?

A typical subarachnoid block is performed in the lumbar region below


the level of spinal cord i.e. L 3-4 in children and L2-3 in adults.

Q: WHICH STRUCTURES ARE PIERCED WHILE PERFORMING


THE BLOCK?
Following structures are pierced from behind forward:-

56

i)
ii)
iii)
iv)
v)
vi)

Skin and subcutaneous tissues


Supraspinous ligament
Interspinous ligament
Ligamentum Flavum
Durmamater
Arachnoid mater

Fig: EQUIPMENT FOR EPIDURAL AND SUBARACHNOID ANAESTHESIA.

Q: WHAT
ARE
ANAESTHESIA?

THE

COMPLICATIONS

OF

SPINAL

57

Complications are:i)
Hypotension.
ii)
Post dural puncture headache (PDPH).
iii)
Nausea and vomiting.
iv)
Meningitis.
v)
Urinary retention.

Q:

WHICH AGENTS ARE COMMONLY USED?

Agents commonly used are:i)


Inj. Lignocaine 2%.
ii)
Inj. Bupivacaine 0.5%.
iii)
Inj. Bupivacaine 0.75%.

Q: WHAT ARE
ANAESTHESIA?

THE

CONTRAINDICATIONS

OF

SPINAL

Contraindications are:i)
ii)
iii)
iv)
v)
vi)
vii)

Patients disapproval.
Infection at the injection site.
Increased intracranial pressure.
Coagulopathy.
Meningitis.
Hypovolaemia and Hypotension.
Valvular heart disease.

EPIDURAL ANAESTHESIA
Q:

WHAT IS EPIDURAL ANAESTHESIA?

In Epidural Anaesthesia local anaesthetics are injected in the epidural


space. The epidural space lies just outside the dural sac, where there
is a negative pressure. The structures pierced by epidural needle: Skin.
Subcutaneous fat.
Supraspinous ligament.
Interspinous ligament.
Ligamentum Flavum, and then is the epidural space.

Q:

DESCRIBE THE ANATOMY OF EPIDURAL SPACE?

Boundaries:

58

Superiorly: closed at foramen magnum.


Inferiorly: closed at sacro-cocccygeal membrane.
Anteriorly: posterior longitudinal ligaments, vertebral
bodies.
Posteriorly: vertebral lamina and Ligamentum Flavum.
Laterally: open, pedicles and intervertebral foramina
Shape:
Triangular, with apex posteriorly.
Contents: Veins, arteries, fat, lymphatic, nerve roots and dural cuffs.

Fig:

LUMBAR EPIDURAL ANAESTHESIA. Notice how the anaesthetists right hand rests against the patients
back to support the needle.

Q: DIFFERENTIATE
ANAESTHESIA?

BETWEEN

SPINAL

AND

EPIDURAL

In spinal anaesthetic: A small amount of local anaesthetic drug is placed directly in the
CSF producing a total neural blockade caudal to the injection site.
It gives rapid, dense and predictable anaesthetic effect.
In epidural anaesthesia:59

Ten-fold increase in dose of local anaesthetic (in comparison to


spinal) is required to fill the potential epidural space.
The onset is slower.
The anaesthesia is segmental i.e. a band of anaesthesia is
produced extending above and below the injection site.

Advantages of epidural anaesthesia:


Epidural anaesthesia causes less hypotension as compared to
spinal anaesthesia. Catheter can be introduced inside the epidural
space and drugs can be given repeatedly for post-operative pain
relief.
Epidural analgesia is a popular technique for painless delivery.
Epidural anaesthesia can be applied at any level of vertebral
column by expert hands.

60

Fig:

THE ANATOMY OF EPIDURAL AND SUBARACHNOID ANAESTHESIA. A, the anatomy for


lumbar puncture with a patient in the sitting position. B, with the patient in the lying position.
The line between his iliac crests passes between his 3rd and 4th lumbar spines. C and epidural
needle goes first through his interspinous ligament and then through his ligamentum flavum
before it reaches his extradural space. In this figure his interspinous ligament has been
dissected away in the segment through which the needle is passing. For subarachnoid
anaesthesia the needle goes further on through his dura and arachnoid mater into his
subarachnoid space, which is filled with CSF.

Q:

HOW EPIDURAL SPACE IS IDENTIFIED?

61

Tuohys needle is passed in the intervertebral space, while passing


through the skin, supraspinous ligament, interspinous ligament and
ligamentum flavum.
Two methods are applied for identification of epidural space.
Loss Of Resistance (LOR) method, using syringe.
Hanging drop method.

Q: WHAT ARE THE CONTRAINDICATIONS OF EPIDURAL


ANAESTHESIA?
Following are the contraindications of epidural anaesthesia: Patients refusal.
Sepsis with haemodynamic instability.
Uncorrected hypovolaemia.
Coagulopathy.

Q: WHAT ARE
ANAESTHESIA?

THE

COMPLICATIONS

OF

EPIDURAL

These are: Hypotension, which can be prevented by fluid preload.


Intravascular injection of local anaesthetic.
Dural puncture and total spinal anaesthesia.
Epidural haematoma.

CAUDAL ANAESTHESIA
Q:

DEFINE CAUDAL ANAESTHESIA?

The sacral epidural is called caudal anaesthesia. In this block local


anaesthetic is injected through sacral hiatus into the epidural space.
The caudal space is the sacral component of the epidural space, and
access is through the sacral hiatus, a midline defect of caudal most
aspect of the sacrum. The space is covered by sacrococccygeal
ligament.

62

Fig:

CAUDAL EPIDURAL ANAESTHESIA. A, the position of the needle in relation to the sacrum. B, the patient
ready for the anaesthetic with a pillow under his pubis. C, making a triangle with the anatomical landmarks. D,
injecting.

63

Q:

Q:

WHAT ARE ITS INDICATIONS?


Surgical and obstetric procedures involving perineum and sacral
distributions, such as anorectal region.
Postoperative pain relief for operations on the lower extremities,
perineum, male genitals and lower abdomen.

WHAT ARE ITS CONTRAINDICATIONS?

Absolute
Sepsis.
Bacteremia.
Skin infection at injection site.
Severe hypovolaemia.
Coagulopathy.
Therapeutic anticoagulation.
Increased intracranial pressure.
Lack of consent.
Sacral decubitus ulcers.
Relative

Q:

Peripheral neuropathy.
Mini-dose heparin.
Aspirin or other antiplatelet drugs.
Certain cardiac lesions.
Psychologic or emotional instability.
Morbid obesity.
Prolonged surgery.
Surgery of uncertain duration.

WHAT ARE THE COMPLICATIONS?


Pain on injection.
Backache.
Headache.
Urinary retention.
Vascular injury, Nerve injury.
Rarely in obstetric practice, injury to fetal head when placing the
needle.
Infections.

64

Q: HOW WOULD YOU GIVE LOCAL BLOCKS FOR THE MOUTH


AND TEETH?
(Following pages are for dental students. The students may practice
these local blocks under the supervision of their teachers.)
A tooth and its surrounding gum are innervated from three
directions: (1) Its pulp is supplied by a nerve which passes up its root.
The gum on (2) its labial and (3) its lingual sides is innervated
separately. The tooth socket is partly supplied by the nerve that
supplies the root and partly by those that supply the gum. If you are
going to remove a patients tooth painlessly, you will have to
anaesthetize all three sets of nerves.
You can easily anaesthetize a patients labial and lingual gums by
local infiltration, but instead of blocking his palatal gums close to this
teeth, it is easier to block them in his palate. Infiltrating his gums or his
palate will at the same time block the nerves that supply most of the
roots of his teeth. The exceptions are his lower molars and second
premolars. To anaesthetize them you will have to block his inferior
alveolar nerve as it enters his mandibular canal.
A patients inferior alveolar nerve supplies all the teeth of his
lower jaw, so blocking this nerve should make all his lower teeth
completely anaesthetic. Unfortunately, anaesthesia is sometimes
incomplete, because small accessory branches enter the bone through
other foramina and so escape the block. Also, his incisors may not be
completely anaesthetized by a single block, because they are
innervated from both sides.
ANAESTHETIZING THE TEETH
DRUGS AND EQUIPMENT For all methods, use 0.5% bupivacaine, or
2% lignocaine with or without adrenaline, preferably in 2 ml cartridges.
If possible, a 10% lignocaine spray, or 5% lignocaine paste. A dental
cartridge type syringe. If necessary, you can use an ordinary one,
preferably one with a Luer-lok. Use thin needles 0.323 and 42 mm.
A spirit lamp to flame the end of the cartridge which has to be pierced.
A pair of straight nosed pliers, or artery forceps, to remove the
broken end of a needle. A decontaminant, such as 0.5% chlorhexidine.
Forceps and some pledgets of cotton wool.
GENERAL METHOD
Sedate the patient with diazepam 10 to 20mg. Explain to him
what your are going to do. Clean his mucosa with the antiseptic. If
possible, spray his mucosa with 10% lignocaine, or apply it as a 5%
paste.

65

After a few seconds stretch his mucous membrane at the site of


the injection and quickly pierce it with the bevel of the needle parallel
to the bone. Inject quickly there is nothing more painful than a local
dental anaesthetic given slowly.
Once you are through his mucosa, you can pause a little while
you find the landmarks. When your needle is in the right position,
inject. You cannot aspirate with a dental cartridge.
Test for analgesia. If you are going to fill a patients tooth, drill its
exposed dentine.. Before pulling it out, test the sensitivity of the gum
around it.

66

Fig:

BLOCKING THE LINGUAL AND INFERIOR ALVEOLAR NERVES. A, is an injection which is too lateral
and B is one which is too medial. X, is the initial position for the syringe, and Y, its final position. C, is the position of
your fingers feeling the ascending ramus of the patients mandible. D, is the position to aim for , midway between your
two fingers.

LOCAL INFILTRATION
FOR ALL UPPER TEETH, THE LOWER INCISORS AND CANINES,
AND ALL DECIDUOUS TEETH Infiltrate the solution outside the
periosteum, near the apex of the tooth. This is where its nerves enter
the bone, so this is your target.
Labially is his upper jaw. Inject at the reflection of the mucous
membrane where it forms the base of the sulcus, as in A, Fig. Inject 12ml of solution, or about half a cartridge. The tip of your needle should
come to lie opposite the tip of the root of the tooth you are going to
extract. For front teeth insert the needle in line with the tooth. This is
impossible with molars, so, if you want to anaesthetize a patients third
molar, insert the needle over his second molar, and aim it obliquely so
that its point comes to lie over the root of his third. If you move the
point of the needle fanwise, as in D, very carefully, you can
anaesthetize 2 or 3 teeth without removing it.
When you inject his upper molars (D), feel the gum on the outer
surface of his upper back teeth. The crest of bone jutting down from
above is his infrazygomatic crest. Insert your needle immediately
behind this crest, distal to his second molar.
Push your needle in 2 cm, as far as it will go, and inject 2 ml of
solution. Move it as fanwise as you inject. This is also called a
tuberosity block.
Palatally in the upper jaw Inject at the points marked X
about 1 cm from the tooth half way between the edge of the gum, and
the mid line, as in B, Fig. This is a shallow injection because his palate
lies close below a patients mucous membrane. Inject just enough
solution to make his gum go white. You will not be able to inject much,
and you will have to press quite hard.
Labially in a patients lower jaw. Hold his lip out of the way
so that you can see the sulcus clearly. Insert the needle next the
chosen tooth, so that its point lies against the outside of his mandible,
level with the tip of the root. Inject half a cartridge.

67

Lingually in the lower jaw. Insert the needle a short distance


at the point where the mucosa is reflected off the lingual side of his
alveolus, as in A, Fig. You may have to hold his tongue out of the way
to see the floor of his mouth. Inject about a quarter of a cartridge.
There will be a small swelling, which will quickly disappear.
THE LOWER PREMOLARS Labially do a mental block like this:
Pull down the patients lower lip. Use the tip of your index finger to feel
the labial surface of his gum as it turns upwards to join his cheek just
posterior to his first premolar tooth. You should be able to feel his
mental nerve as it comes out of the mental foramen in his mandible.
Inject from behind, as in F Fig. Pull the corner of the patients
mouth out of the way. Tilt the needle medially between his first and
second premolars. Aim to place the needle just outside his mental
foramen. This is half way between his gingival margin and the lower
border of his mandible. As a person gets older, his mandible is
absorbed, so that his mental foramen comes to lie nearer the upper
border of this mandible. Inject 2 ml of solution. If necessary, repeat the
procedure on the other side.
Try not to enter his mental foramen, because you may injure the
vessels that come out of it, and so cause a large haematoma.
Lingually inject his premolars in the same way as for his lower incisors
and canines.
LOWER MOLARS Do an inferior alveolar and lingual nerve block,
as described below.

68

Fig:

INFILTRATING THE LOWER GUMS. A, infiltrating the lingual and B, the labial gum.

RIGHT INFERIOR ALVEOLAR AND LINGUAL NERVE BLOCK:Landmarks The secret of success is to visualize where the patients
mandibular foramen is, and to aim the tip of a 42 mm needle at it. As
usual, the details are all important.
Adjust the headrest, so that when the patients mouth is wide
open, the occlusal plane of his mandible is horizontal, as in D, Fig.
When you are learning, use a dental stick dipped in gentian violet to
draw a line QR on the mucous membrane of the inside of his cheek in
the line of the occlusal surfaces of his lower teeth. If he has a denture,
draw it with this in place. If marking it makes him retch, anaesthetize
his mucosa first.
69

Feel the anterior and posterior borders of the ascending ramus of his
mandible between the thumb and index finger of your left hand, as in
C. Make sure that your index finger is as far up his mandible as it will
go. The tips of your fingers should lie at either end of line QR. Aim at
the mid point between them usually 2 cm behind point R. Rest the
syringe on the occlusal surfaces of the teeth.

Fig:

INFILTRATION ANAESTHESIA FOR THE TEETH. A, when you infiltrate a patients gum, put
the needle into his buccal sulcus, make the bevel face his periosteum and inject just outside it.
B, to anaesthetize his palatal gums inject at the point marked X. C, infiltrating the palatal
gum of his first molar. D, infiltrating the buccal aspect of his third molar (tuberosity block). E,
infiltrating the gum of his lateral incisor. F, blocking his mental nerve. His mental foramen lies
on a vertical line between his 4th and 5th teeth, and in a young person is half way up his
mandible.

70

The block Now you know the landmarks, put your left index finger into
the patients mouth, above his lower third molar, as in the upper
diagram in Fig. you will feel a depression in the bone immediately
above and behind it (retromolar fossa). Behind this you will find a ridge
(the oblique line), on the inner surface of his mandible.
Ask him to open his mouth even wider.
Insert the needle, as described above, immediately medial to the
oblique line, 1 cm above the patients third molar. At first, place the
syringe in the line of the body of his mandible. This is position X. As
you push the needle in 2 cm, move the barrel of the syringe across his
teeth, so that it lies over his opposite premolar. This is position Y. As
you move the needle, keep it in contact with his teeth all the time. If he
has no teeth, keep it carefully horizontal in his mouth. As you do so,
you will feel the needle pass through the buccinator muscle. As it goes
through, inject 0.5 ml of solution.
Push the needle 2.5 cm further in until it reaches the medial
surface of the ramus of his mandible. Inject 2.5 ml here to block his
inferior alveolar nerve. If you reach bone at a lesser depth, your needle
is too far lateral (needle A in Fig). If you feel no bone, it is too far
medial (needle B).
After you have withdrawn the needle, inject the last 1 ml of
solution into his buccal sulcus, just above the crown of his third molar
tooth. This will block his buccal nerve, as it lies on the inner surface of
his buccinator muscle.
The latent period lasts 10 minutes. The whole of one side of the
patients face will feel heavy, and his lower lip will feel dead on that
side.
If anaesthesia of his canine is not complete, infiltrate his gum, or
block his mental nerve.
CAUTION! (1) Dont push the needle completely into the patients
tissues, if it breaks you will have great difficulty removing it. (2) Before
starting to extract a tooth, press the beak of the forceps hard on both
sides of the tooth. If he feels pain, give him another injection.

71

Fig: INSERTING THE NEEDLE TO BLOCK THE INFERIOR ALVELAR NERVE. Notice the position of the point
of the needle.

72

Lecture 7
FLUID MANAGEMENT
Q: WHY IS
NECESSARY?

EVALUATION

OF

INTRAVENOUS

VOLUME

Most of the patients undergoing surgical procedures require venous


access and intravenous fluid therapy. The assessment of intravenous
volume and replacement of any fluid or electrolyte deficits and ongoing
losses is very important. Errors in fluid management or transfusion
may result in considerable morbidity and mortality.

Q: WHAT
ARE
THE
INTRAVENOUS VOLUME?

STEPS

OF

EVALUATION

OF

The following are the steps of evaluation of intravenous fluid volume:1.


HISTORY:History of abnormal fluid loss.
History of NPO hours.
2.
PHYSICAL EXAMINATION:Physical examination is important to assess whether the patient is
suffering from hypovolemia or hypervolemia.
Signs of hypovolemia:Skin turgor is lost.
Dry mucous membranes.
Dull sensorium.
Weak peripheral pulses.
Increased pulse rate.
Decreased blood pressure.
Orthostatic changes in pulse and blood pressure.
Urine output less than 0.5 ml/kg/hr.
Late signs of hypervolemia are:Tachycardia.
Pulmonary rales.
Cyanosis.
Wheezing.
Pink frothy sputum.
Presacral or pretibial pitting edema.
Increased urinary flow.
3.

HAEMODYNAMIC MEASUREMENT:73

Body fluid status can be measured by measurement of


CVP (Central Venous Pressure).
PCWP (Pulmonary Capillary Wedge Pressure).
4.
LABORATORY EVALUATION:Blood:Serum haematocrit ---- increases with dehydration.
Arterial pH ---- metabolic acidosis in dehydration.
Serum sodium ---- increases in dehydration.
Serum urea ---- increases in dehydration.
Urea to creatinine ratio ---- 10:1.
Urine:Urine specific gravity ---- 1.010.
Urine osmolarity > 650 m. osmol/kg.
Urinary sodium concentration < 20meq/l.
Urinary chloride concentration is decreased.

Q:

WHAT ARE THE TYPES OF INTRAVENOUS FLUIDS?

There are two types of intravenous fluid:Crystalloids.


Colloids.

Q:

WHAT ARE CRYSTALLOIDS?

Crystalloids are aqueous solutions of low molecular weight with or


without glucose. When given they rapidly equilibrate and distribute
throughout the entire extracellular fluid space. Crystalloids are equally
effective when given in sufficient amounts in restoring intravascular
volume, usually 3-4 times the volume of blood lost.

Q:

WHAT ARE THE TYPES OF CRYSTALLOIDS?

A wide variety of solutions are available. e.g.


0.9% normal saline.
Hartmans solution.
5% dextrose solution.
The solutions should be chosen according to the type of fluid loss being
replaced.
1.
maintenance type solutions:Losses primarily due to water loss should be replaced with
hypotonic solutions.
2.
Replacement type solutions:-

74

Losses that involve both water and electrolyte deficit should


be replaced with isotonic electrolyte solutions.

Q:

WHAT ARE COLLOID SOLUTIONS?

Colloid solutions contain high molecular weight proteins or large


glucose polymers usually in normal saline. Colloid solutions maintain
osmotic pressure and mostly remain intravascular for 3-6 hours.
Severe intravascular fluid deficits can be more rapidly corrected using
colloid solutions.

Q: WHAT
COLLOIDS?

Q:

Q:

ARE

THE

INDICATIONS

FOR

THE

USE

OF

Fluid resuscitation in patients with severe intravascular fluid


deficit.
Fluid resuscitation in patients with severe hypoalbunemia.
Conditions associated with severe protein loss such as burn.

WHAT ARE THE TYPES OF COLLOIDS?


Blood derived colloids.
Albumin 5% and 25% solutions.
Plasma protein fraction 5%.
Dextrose starches.
Dextran.
Starch solution.
Gelatins.

WHAT ARE THE SIDE EFFECTS OF DEXTRAN?


Antiplatelets effect.
May interfere with blood typing.
May prolong bleeding time.
May cause renal failure.
Mild to severe anaphylactic reactions can occur.

Q: WHAT
THERAPY?

ARE

THE

STEPS

OF

PERIOPERATIVE

FLUID

This includes normal maintenance requirements:This means replacement of normal losses such as: Urine formation.
75

GIT secretions.
Sweating.
Insensible loss from skin and respiratory tract.
This is a hypotonic loss and is replaced with solutions such as.
5% Dextrose water with saline.
5% Dextrose water without saline.

Estimation of normal maintenance requirements:1st 10 kg ------------------ 4 ml/kg/hr


Next 10 kg ---------------- 2 ml/kg/hr
For each kg above 20 kg
1ml/kg/hr
Pre-existing deficits
Fluid deficit due to period of fasting before surgery: Normal maintenance rate X duration of fast
Abnormal fluid loses
Pre operative bleeding
Vomiting
Diuresis
Diarrhoea
Occult loses
Third spacing
Ascites
Increased insensible losses due to hyperventilation, fever and
sweating
To replace pre-existing deficits fluid should be similar in
composition to the fluid lost.
Surgical fluid losses:Blood loss is estimated;
Suction container.
Visual estimation of blood loss.
Sponges 4 x 4 --- 10 ml.
Laparotomy pads ---- 100 150 ml.
Blood loss can be measured by weighing sponges before and after
use and by serial haematocrit and hemoglobin concentrations.
Other fluid losses
Evaporation.

Internal distribution.
Interstitial space (third spacing).
76

Across serosal surface (Ascites).


Into bowel lumen.
Significant loss of lymphatic fluid may occur during extensive
retroperitoneal dissection.

Q: What is Daily Fluid requirement Post-operatively?


Daily Fluid Requirement Post-operative

Adult Patient
According to Body wt.
First 10kg 4 kg/hr
(4 10) = 40 ml/hr
Next 10kg 2 kg/hr
(2 10) = 20 ml/hr
Next 10kg 1 kg/hr
(1 10) = 10 ml/hr
For example 70 kg wt patient
First 10kg 40 ml/hr
Next 10 kg 20ml/hr
Next 10 kg 50ml/hr
Total 110ml/hr
Adult open heart surgery
1st day 1ml/kg/hr
2nd day 1.5 ml/kg/hr
Paediatric Cases
First 10kg 4ml/kg/hr
Next 10kg 2ml/kg/hr
Then Next 1ml/kg/hr

BLOOD TRANSFUSION
Q: WHAT ARE THE INDICATIONS OF BLOOD TRANSFUSION
IN A SURGICAL PATIENT?

77

Hb % less than 7 gm/dl and / or


Haematocrit less than 21% (in a patient undergoing elective
surgery)
10-20% blood volume loss

Q: WHAT
ARE
TRANSFUSION?
1.

2.
3.
4.
5.

6.
7.
8.
9.
10.
11.

Q:

THE

COMPLICATIONS

OF

BLOOD

Infection
a. Viruses
Hepatitis C
Hepatitis B
HIV
Cytomegalovirus
b. Bacteria
Syphilis
c. Protozoa:
Malaria, toxoplasmosis
ABO incompatibility
Anaphylaxis
Adverse transfusion reaction
Massive transfusion problems
a. Hyperkalemia: high K+ levels in blood, may be if
transfusion more than 1.5 ml/kg/min.
b. Hypocalcaemia: citrate chelates ionized calcium
c. Hypomagnesaemia
d. Acid-base derangements: initial acidosis becomes alkalosis
as citrate metabolized to bicarbonate
e. Hypothermia
Oxygen dissociation curve shifts to the left, so less oxygen is
delivered to the tissue
Micro embolism
Hyperglycaemia
Dilutional thrombocytopenia
Dilutional coagulopathy
Transfusion-related acute lung injury

HOW IS BLOOD STORED?

78

Red cells last well in refrigerated (4-6 C) stored blood. More than 70%
survive 24 hrs after transfusion.
Clotting factors deteriorate
progressively after 24 hrs storage. Citrate-phosphate-dextrose (CPD)
blood contains no functional platelets after 48h. citrate-phosphate
dextrose blood plus adenine preserves its adenosine triphosphate (ATP)
and 2,3-DPG levels for up to 2 weeks with slow fall thereafter and is
stored for up to 35 days.

Q:

WHAT ARE VARIOUS BLOOD PRODUCTS AVAILABLE?


1.
2.
3.
4.
5.
6.
7.
8.
9.
10.
11.

Q:

Saline adenine glucose mannitol blood


Whole blood
Frozen blood
Plasma
Human albumin
Human fibrinogen
Cryoprecipitate
Platelets
Factor VIII concentrate
Factor IX
Washed red cells

WHAT ARE THE SIGNS OF INCOMPATIBLE TRANSFUSION?

In the conscious
1.
2.
3.
4.
5.
6.
7.
8.
9.
10.
11.
12.
13.

Headache
Precordial or lumbar pain
Urticaria or pruritus
Burning in limbs
Bronchospasm
Dyspnoea
Tachycardia
Restlessness
Suffused face
Nausea and vomiting
Pyrexia and rigors
Circulatory collapse
Later, haemoglobinaemia, haemoglobinuria and oliguria.

Under anaesthesia
Sometimes not easy to distinguish from the effects of hemorrhage
itself, especially during rapid transfusion > 100 ml/min.
79

Immediate
1.
Rapid severe and progressive hypotension
2.
Tachycardia
3.
General oozing from wound
4.
Urticarial rash
5.
Bronchospasm, raising airway pressures
positive pressure ventilation.

on

intermittent

Late
Jaundice and oliguria in 5-10% of these patients. It strongly
resembles anaphylactic reaction and treatment is similar.

Q: WHAT IS THE MANAGEMENT OF SEVERE TRANSFUSION


REACTION?
The blood unit should be rechecked against blood slip and patients
identity bracelet (Patients Chart). Blood should be drawn to identify
haemoglobin and coagulation tests.
1.
2.
3.
4.
5.
6.
7.
8.
9.
10.

Stop the transfusion.


Support blood pressure with intravenous colloids or
crystalloids and ionotropes or vasoconstrictors if needed.
100% O2.
Induce a diuresis with mannitol 50 g or frusemide 10-20 mg.
Check acid-base balance and electrolyte.
Exchange transfusion in the desperate case.
High dose of steroids may be useful.
Antihistamines may be indicated in the early stages but may
increase hypotension.
Where DIC is occurring, coagulation factors and platelets need
to be replaced.
Transfer to high-dependency unit or intensive therapy unit.

80

Lecture 8
ACID-BASE & ELECTROLYTE BALANCE
ACID BASE BALANCE

Q:

DEFINE THE FOLLOWING:

pH, ACID, BASE, ACIDOSIS, ACIDEMIA, ALKALOSIS,


ALKALEMIA?
pH:
It is the negative logarithmic value of H+ concentration
pH= 1/[H+]
Acids:
Substances that produce H+ when dissolved in water.
Acid-Base disorders:
Normal pH of body fluids is 7.36-7.44
Acidosis:
Process that cause acids to accumulate
Acidemia:
Present when pH < 7.36
Alkalosis:
Process that cause base to accumulate
Alkalemia:
Present when pH > 7.44

Q: WHAT ARE THE DIFFERENT ACID-BASE DISORDERS?


EXPLAIN.
Primary acid-base disorders are:Respiratory: if primary disturbance involves CO2
Metabolic: if primary disturbance is in HCO3 concentration.
METABOLIC ACIDOSIS
Characterized by decreased HCO3 and a variable degree of acidemia.
Respiratory compensation is by hyperventilation thus washing out CO2.

81

Clinical features:Decreased cardiac output


Pulmonary hypertension
Arrhythmias
Kussmaul breathing
Hyperkalaemia
Causes
Overproduction of acids:Diabetic ketoacidosis
Lactic acidosis
Exogenous acid:Salicylates
Reduced excretion:Renal failure
Treatment
Identification of cause and its treatment. If pH < 7.2, measures taken
to restore pH. Sodium bicarbonate can be used for restoration of pH
towards normal.
Bicarbonate requirement: body wt. (in kg) x base deficit x 0.3
METABOLIC ALKALOSIS
Characterized by primary increase in HCO3 and a variable degree of
alkalemia. Compensatory respiratory hypoventilation is very limited
and not effective. For diagnostic point of view, metabolic alkalosis is
divided into:
Chloride responsive
Chloride resistant
Chloride responsive:
Loss of acid
Vomiting
N/G suction
Gastrocolic fistula
Chloride depletion
Diarrhoea
Diuretic abuse
Excessive alkalies
NaHCO3 administration
Chloride resistance
Primary or secondary hyperaldosteronism
Cushing syndrome
Severe hypokalemia
82

Treatment
In chloride responsive alkalosis, administration of saline causes volume
expansion and results in excretion of excess bicarbonate; if K + is
required, it should be given as KCl. In patients to whom volume is
administered, the use of acetazolamide results in renal loss of HCO 3
and an improvement in pH.
In life threatening metabolic alkalosis, rapid correction is necessary
and may be achieved by administration of H+ in the form of dilute HCl.
Acid is given as 0.1 normal HCl in glucose 5 % at a rate no more than
0.2 mmol/kg/hr.
RESPIRATORY ACIDOSIS
Characterized by an increase in CO2, which results in acidemia
proportional to degree of hypercapnia. Compensation is through
kidneys, which excrete acid.
Clinical features
Usually hypoxemia and manifestations of underlying disease dominate
the clinical picture but hypercapnia per se may result in coma, raised
ICP and hyper dynamic CVS resulting, from release of catecholamine.

Causes:CNS
Drug over dosage
Trauma
Tumor
PNS
Polyneuropathy
Myasthenia gravis
Poliomyelitis
Tetanus
Primary pulmonary disease
Airway obstruction:Asthma
Parenchymal disease:ARDS
Loss of mechanical integrity:Flail chest
Treatment
Treatment of underlying cause and mechanical ventilation if required.

RESPIRATORY ALKALOSIS
Primary decrease PaCO2, which increase pH above 7.44
Clinical features:83

Lightheadedness
Confusion
Seizures
Circumoral paraesthesia
Hyperreflexia
Tetany
Causes
Hyperventilation voluntarily or hysteria
Pain, anxiety
Specific conditions
CVS disease
Meningitis
Tumor
Trauma
Respiratory disease
Pneumonia
Pulmonary embolism
Shock
Cardiogenic
Hypovolaemic
Treatment
Treatment of underlying cause

ELECTROLYTE BALANCE
SODIUM BALANCE
Q:

WHAT IS THE NORMAL SODIUM BALANCE OF THE BODY?

Daily ingestion amounts to 50-300 mmol. Losses in sweat and faces


are minimal (approx. 10 mmol/day) and final adjustment are made by
kidney. Urine Na may be as low as 2 mmol/day during salt restriction
or may exceed 700 mmol/day after salt loading. Sodium balance is
related intimately to ECFV and water balance.

Q:

DEFINE HYPERNATREMIA?

Hypernatremia is defined as a plasma sodium concentration of more


than 150 mmol/litre. It may result from pure water loss, hypotonic fluid
loss or salt gain. In the first two conditions, ECFV is reduced, whereas
salt gain is associated with an expanded ECFV.

84

Q:

WHAT ARE THE VARIOUS CAUSES OF HYPERNATREMIA?

Pure water depletion


External loss
postoperative)
Renal loss
Hypotonic fluid loss
External loss
Renal loss
Salt gain

Failure of water intake (coma, elderly,


Mucocutaneous loss
Fever, hyperventilation, thyrotoxicosis
Diabetes inspidus (cranial, nephrogenic)
Chronic renal failure
Gastrointestinal (vomiting, diarrhea)
Skin (excessive sweating)
Osmotic diuresis (glucose, urea, mannitol)
latrogenic (NaHCO3, hypertonic saline)
Salt ingestion
Steroid excess

Q:

WHAT ARE THE CONSEQUENCES OF HYPERNATREMIA?

Reduction in cell volume and water content of the brain.


Increased permeability and even rupture of the capillaries in the
brain and subarachnoid space.
Pyrexia.
Nausea, vomiting.
Convulsions.
Coma.
Focal neurological syndrome.

Q:

WHAT IS THE TREATMENT OF HYPERNATREMIA?

For hypernatremic patients without volume depletion 5% glucose is


sufficient to correct the water deficit. Water deficit is calculated as:
Water deficit = (measured [Na +] /140xTBW)-TBW
For hypernatremic patients with hypovolaemia, isotonic saline is the
initial treatment of choice. Once volume depletion is corrected then
further repair of any fluid deficit can be accomplished with hypotonic
fluids.

Q:

DEFINE HYPONATREMIA?

This is defined as a plasma sodium concentration of less than 135


mmol/litre.

85

Q:

WHAT ARE THE CAUSES OF HYPONATRAEMIA?


HYPONATREMIA

EVALUATION OF EXTRACELLULAR FLUID VOLUME

HYPOVOLAEMIA
NORMOVOLEMIA

HYPERVOLAEMIA

(EDEMA)

osmolarity

plasma

Renal
Loss

EXTRA RENAL
LOSS

Diuretic
abuse
Hypoadrenalis
m
- Salt losing
nephropathy
Renal tubular
- Acidosis
-

Diarrhoea
Vomiting
Third
space losses

Congestive cardiac
failure

Urine sodium
< 15mmol/l

Urine sodium
> 20mmol/l

NORMAL
Psuedohypo
natraemia

Urine sodium
<20mmol/l

DEPLETIONAL SYNDROMES
SALINE REQUIRED
*

LOW
- SIADH*
- SIIVT**
- Drugs
- Hypothyroid
- Stress(postop
)
- Renal failure

Cirrhosis
Nephrotic Syndrome

Urine Sodium
variable

DILUTIONAL SYNDROMES
FLUID RESTRICTION REQUIRED

Syndrome to inappropriate ADH Secretion

86

**

Syndrome of inappropriate intravenous therapy

Q:

WHAT ARE THE CONSEQUENCES OF HYPONATRAEMIA?

Intracellular overhydration
Cerebral odema
Raised intracranial pressure
Nausea, Vomiting, Delerium, Convulsions, Coma

Q:

WHAT IS THE TREATMENT OF HYPONATRAEMIA?

Acute symptomatic hyponatraemia is a medical emergency and


requires prompt intervention using hypertonic saline. The amount of
sodium needed to cause the desired correction in the plasma sodium
can be calculated as follows:
Na+ required (mmol) = TBW x (desired [Na +] measured [Na+])

POTASSIUM BALANCE
Q:

WHAT IS THE NORMAL POTASSIUM BALANCE?

The normal daily intake of potassium is 50-200 mmol. Minimal


amounts are lost via the skin and feaces; the kidney is the primary
regulator.

Q:

DEFINE HYPOKALEMIA?

This is defined as a plasma K+ concentration of less than 3.5mmol/l.

Q:

WHAT ARE THE CAUSES OF HYPOKALEMIA?

Causes
Reduced intake
Tissue
redistribution
Increased loss
Gastrointestinal
(Urine K+ <
20mmol/l)
Renal Loss

Comments
Usually only contributory
Insulin therapy, alkalaemia, B2- adrenergic agonists,
familial periodic paralysis, vitamin B12 therapy.
Diarrhoea, vomiting, fistulae, nasogastric suction,
colonic villous adenoma.
Diuretic therapy, primary or secondary
hyperaldosteronism, Malignant hypertension, renal
artery stenosis (high renin), Renal tubular acidosis,
hypomagnesemia, renal failure.
87

Q:

Q:

WHAT ARE THE SYMPTOMS OF HYPOKALEMIA?


Anorexia and nausea.
Muscle weakness and paralytic ileus.
Delayed repolarization with ST segment depression.
Reduced height of T wave.
Widened QRS complex.

WHAT IS TREATMENT OF HYPOKALEMIA?

As a general rule, a reduction in plasma K + concentration by 1 mmol/l


reflects a total body k+ deficit of approximately 100 mmol.
K+
supplements may be given orally or I/V. The maximum infusion rate
should not exceed 0.5 mmol/kg/h to allow equilibrium with intracellular
compartment.

Q:

DEFINE HYPERKALEMIA

This is defined as a plasma K+ concentration exceeding 5 mmol/l.


Fictitious
(pseudohyperkalaemia)

Impaired excretion

Tissue redistribution

Excessive intake

Q: WHAT ARE
HYPERKALAEMIA?

In vitro haemolysis
Thrombocytosis
Leucocytosis
Tourniquet
Exercise
Renal failure
Acute or chronic hyperaldosteronism
Addisons disease
K+ - sparing diuretics
Indomethacin
Tissue damage (burns, trauma)
Tumor necrosis
Massive intravascular haemolysis
Suxamethonium
blood transfusion
Excessive i.v. administration

THE

EFFECTS

AND

TREATMENT

OF

88

EFFECTS
Skeletal muscle weakness.
Peaked T wave.
Shortened QT interval.
Ventricle fibrillation.
Asystole.
TREATMENT
Calcium gluconate 10% i/v. (0.5 ml/kg to maximum of 20 ml) given
over 5 min. no change in plasma [K+]. Effect immediate but
transient.
Glucose 50 gm (0.5-1.0g/kg) plus insulin 20 units (0.3 unit/kg) as
single i.v. bolus dose.
Sodium bicarbonate 1.5 2.0 mmol/kg i.v. over 5-10 min.
Calcium resonium 15 g p.o. or 30 p.r. 8-hourly.
Peritoneal or haemodialysis.

Formula to calculate requirement of K+ in a patient.


K+ Requires= 0.4 Body wt in kg (5 Lab Value)

89

Lecture 9

BLOOD GASES, PULSE OXIMETRY AND CAPNOGRAPHY


BLOOD GASES
Q:

WHAT ARE THE NORMAL BLOOD GAS VALUES?

The normal values for different parameters considered in the blood


gases are:
Arterial
Venous
7.35-7.45
pH
7.31-7.41
80-100 mm Hg
pO2
30-40 mm Hg
35-45 mm Hg
pCO2
41-51 mm Hg
21-25 mEq/L
HCO3
22-29 mEq/L
95% - 99%
O2 sat
60% - 85%
- 2 to + 2
BE
0 to +4
In a patient > 60 years old, PaCO2 is equal to 80 mm Hg minus 1 mm
Hg for every year over 60.

Q: HOW DO YOU INTERPRET ARTERIAL BLOOD GASES


(ABGs) VALUES?
1.

2.

3.

4.
5.

Check pH
= Alkalosis
= Acidosis
Check pCO2
= CO2 retention (hypoventilation); respiratory acidosis or
compensating for
metabolic alkalosis.
= CO2
blown off (hyperventilation); respiratory alkalosis or
compensating
for metabolic acidosis.
Check HCO3
= Nonvolatile acid is lost; HCO3 gained (metabolic alkalosis or
compensating
for respiratory acidosis)
= Nonvolatile acid is added; HCO3 is lost (metabolic acidosis or
compensating for respiratory alkalosis)
Determine imbalance
Determine if compensation exists

90

Q: WHAT CHANGES IN PARAMETERS WILL HELP YOU TO


DETERMINE IMBALANCE IN ABGs?
If
pH

pH

If
pH

pH

If
pCO2
pCO 2
If
pCO2
pCO 2

Q:

and pCO2
or
and pCO2

and HCO3
or
and HCO3

and HCO3
or
and HCO3

and HCO3
or
and HCO3

Then respiratory disorder

Then metabolic disorder

Then compensation is occurring

Then mixed imbalance

HOW WOULD YOU DETERMINE COMPENSATION?

To determine the compensation for respiratory or metabolic disorders,


pH and PaCO2 are the basic parameters. The normal range of
PH = 7.30 7.50
PaCO2 = 30-50 mm Hg
So remember range. 30-50
For compensation of respiratory alkalosis / acidosis: check the pH
For compensation of metabolic alkalosis / acidosis: check pH and
PaCO2
Now
- If PaCO2 is normal and pH is outside the range the disorder is
uncompensated
- If PaCO2 is increased or decreased and pH is outside the range
the disorder is partially compensated
- If PaCO2 is increased or decreased and pH is within range the
disorder is fully compensated.

91

See for example.


If ph = 7.26, PaCO2= 56, HCO3 = 24 then respiratory acidosis is
uncompensated
If ph = 7.38 PaCO2= 76, HCO3 = 24 then respiratory acidosis is
compensated
If ph=7.2, PaCO2=40, HCO3 =9 then metabolic acidosis is either acute
or uncompensated
If ph = 7.36 PaCO2= 25, HCO3 = 15 then metabolic acidosis is
compensated

PULSE OXIMETRY
Q:

WHAT IS PULSE OXIMETRY?

This is a non-invasive measurement of the arterial blood oxygen


saturation at the level of the arterioles. A continuous display of the
oxygenation is achieved by a simple, accurate and rapid method.
Pulse oximetry has proved to be a powerful monitoring tool in the
operation theatre, recovery wards, and intensive care units, general
wards and during transport of critically ill patients.

Q:

WHAT ARE THE COMPONENTS OF A PULSE OXIMETER?


1. A probe positioned on the finger, toe, ear lobe or nose. The light
emitting diodes (LEDs) producing beams at red and infrared
frequencies on one side and a sensitive photodetector on the
other
side.
The
LEDs
operate
in
sequence.
2. The case, which houses the microprocessor. There is a display of
the oxygen saturation, pulse rate and a plethysmographic
waveform of the pulse. Alarm systems can be set for a low
saturation value and for both high and low pulse rates.

Q: WHAT IS THE MECHANISM OF ACTION OF PULSE


OXIMETER?
1. The oxygen saturation is estimated by measuring the
transmission of light, through a pulsatile vascular bed (e.g.,
finger).
2. The amount of light transmitted depends on many factors. The
light absorbed by non-pulsatile tissues (e.g., skin soft tissues,
bone and venous blood) is constant (DC). The non-constant
absorption (AC) is the result of arterial blood pulsation. The
sensitive photodetector generates a voltage proportional to the
transmitted light.
92

3. The microprocessor is programmed to mathematically analyze


both the DC and AC components. The result is the arterial
saturation.

Q:

WHAT ARE THE FACTORS THAT AFFECT ITS WORKING?


1. It is accurate in the range of 70-100%. Below the saturation of
70% readings are extrapolated.
2. Carbon monoxide poisoning (including smoking), coloured nail
varnish, I/V injections of certain dyes and drugs responsible for
the production of methaemoglobinemia are all sources of error.
3. Excessive movement or malposition of the probe is a source of
error.
4. Inaccurate measurements can be caused by venous pulsations.

CAPNOGRAPHY, END TIDAL CO2 CONCENTRATION


Q:

WHAT IS A CAPNOGRAPH?

An instrument, which gives a continuous recording of end-tidal carbon


dioxide concentration, is known as a capnograph.

Q:

HOW IS INFRARED ANALYZER USED FOR THIS PURPOSE?

Gases with molecules that contain at least two dissimilar atoms absorb
radiation in infrared region of spectrum. Using this property, carbon
dioxide concentration can be measured directly and continuously
throughout the respiratory cycle. End-tidal carbon dioxide reflects
accurately the arterial carbon dioxide tension in the individuals with
normal lungs.

Q:

WHAT ARE ITS COMPONENTS?


1. The sample chamber can either be positioned within the
patients gas stream or connected to the distal end of the
breathing system via a sampling tube.
2. A photodetector measures light reaching it from a light source at
the correct infrared wavelength after passing through two
chambers. One acts as a reference whereas the other one is the
sample chamber.

Q:

WHAT IS THE MECHANISM OF ACTION?


93

1. CO2 absorbs the infrared radiation particularly at a wavelength of


4.3 um.
2. The amount of radiation absorbed is proportional to the number
of CO2 molecules (partial pressure of CO 2) present in the
chamber.
3. The electrical output from the detector is proportional to the
partial pressure of CO2 in the chamber.
4. In the same way a beam of light passes through the reference
chamber, which contains air. The absorption detected from the
sample chamber is compared to that in the reference chamber.
This allows the calculation of values.

Q:

WHAT ARE THE TYPES OF CO2 ANALYZERS?


1. Side stream analyzer
2. Main stream analyzer

Q:

WHAT ARE THE USES OF END TIDAL CO2?

Increased end-tidal CO2


Hypoventilation
Rebreathing
Sepsis
Malignant hyperpyrexia
Hyperthermia
Skeletal muscle activity
Decreased end-tidal CO2
Hyperventilation
Pulmonary embolism
Hypoperfusion
Hypometabolism
Hypothermia

Q: WHAT ARE THE PROBLEMS IN THE USE OF INFRARED


ANALYZER?
1. In patients with chronic obstructive airway disease, the
waveform shows a sloping trace and does not reflect end-tidal
CO2.
2. During pediatric anaesthesia, it can be difficult to produce and
interpret end tidal CO2 due to high respiratory rates and small
tidal volumes.
94

3. Dilution of end-tidal CO2 can occur, whenever there are loose


connections or system leaks.
4. Nitrous oxide absorbs infrared light with an absorption spectrum
overlapping that of CO2, thus causing inaccuracy.
5. The absorption of CO2 is increased due to the presence of nitrous
oxide or nitrogen.

95

Lecture 10

ANAESTHESIA AND RELATED DISEASES


HYPERTENSION
Q: HOW WILL YOU DEFINE HYPERTENSION?
Hypertension is characterized by a persistent.
Systolic blood pressure > 140 mm Hg.
Diastolic blood pressure > 90 mm Hg.

Q: CLASSIFY HYPERTENSION.
CLASSIFICATION OF BLOOD PRESSURE FOR ADULTS
Category
Normal
High normal
Hypertension
Stage 1 (mild)
Stage 2 (moderate)
Stage 3 (severe)
Stage 4 (very severe)

Systolic (mm Hg)


<130
130-139

Diastolic (mm Hg)


<85
85-89

140-159
160-179
180-209
>210

90-99
100-109
110-119
>120

Q: HOW DOES HYPERTENSION AFFECT ADMINISTRATION OF


ANAESTHESIA?
Hypertension may lead to ischemia, myocardial infarction, acute LVF,
CVA, haemorrhage, disruption of vascular suture lines.
The three main features of hypertension which directly influence the
administration of anaesthesia to these patients are:

Altered vascular reactivity to drugs due to altered flow


resistance, smooth muscle shortening relationship in arteriole.
Pre-existing coronary artery disease (CAD) with high blood
pressure.
Existence of LVF and renal failure.

Q: WHAT PROBLEMS CAN YOU FACE DURING ANAESTHESIA?


1. Excessive hypotension may occur during induction of anaesthetic
agents:-

96

Give slow I/V injection of anaesthetic agents.


Stabilize the heart with I/V narcotic analgesic like ultra short
acting Fentanyl.
Inhalation of volatile anaesthetics over 5 mins with N2O: Oxygen.
Give lignocaine 1mg/kg IV bolus dose to attenuate reflexes at
intubation.
2. Excessive hypertension during intubation:Intubation leads to excessive sympathetic discharge,
tachycardia, and dysrythmias.
Give lignocaine 1mg/kg body wt.
Short acting beta blockers such as esmolol I/V.
Local spray of cords.
3. Acute hypertension during maintenance:Titrate volatile anaesthetic.
Avoid hypocapnia and hypercapnia.
4. Rebound hypertension during recovery:If diastolic BP <120mmHg, no active measure.
Give proper analgesia and avoid hypoxia.
If diastolic BP >120mm Hg, give anti-hypertensive agents such
as nitroprusside, nitroglycerine.

ASTHMA
Q: DEFINE ASTHMA? WHAT ARE THE FEATURES OF ASTHMA?
Asthma is a disease characterized by an increased responsiveness of
the trachea and bronchi to various stimuli manifested by a widespread
narrowing of the airways that changes in severity either spontaneously
or as a result of therapy.
Salient features are:

Variable wheezing, dyspnoea and cough due to widespread


airway narrowing.

Q: WHAT ARE THE TRIGGERING FACTORS FOR ASTHMA?


1. Anxiety, airway instrumentation, exercise, allergens (dust,
pollens etc.)
2. Thiopentone, muscle relaxants and beta-blockers may cause
bronchoconstriction.
May be due to IgE mediated histamine release.
3. 10% of adult asthmatics are sensitive to aspirin and other
NSAIDs.

97

Q: WHAT ARE THE IMPORTANT HISTORICAL FEATURES OF AN


ASTHMATIC PATIENT?
A careful history allows a close estimate of severity of the disease and
should include the following questions:
1. How and when the patient was first diagnosed?
2. How often the patient has attacks, what typically initiates them,
and how long the illness has lasted?
3. Whether the patient has been treated as an outpatient or
inpatient? If inpatient, ask for details of hospital stay, including
length of admission, requirement of intensive care, and
intubation.
4. What medication the patient takes, including as-needed usage
and over-the counter medications. Has the patient ever taken
steroids?

Q: WHAT PREOPERATIVE TESTS SHOULD BE ORDERED?


Following investigations should be requested:
1.

2.
3.
4.

Routine investigations like blood C/P, urine R/E, serum


electrolytes etc.
Chest X-Ray.
ECG.
Lung function tests; FEV1, FEV1/FVC, ABGs.

Q: WHAT IS THE ANAESTHETIC MANAGEMENT OF ASTHMA?


Anaesthetic management:

Except in emergency, patients with asthma should not be


operated upon until their lungs condition is optimal.
Premedication: Antihistamines are useful.
Regional techniques are preferred.
Volatile anaesthetics are bronchodilator and well tolerated.
Nebulized Salbutamol can be given during the operation.

Asthma may get worse by: I./V Thiopentone


Clumsy inhalational induction
Tracheal intubation

98

Incomplete relaxation
Regurgitation
Avoid beta blockers
(Cardio selective, Metoprolol 2-10 mg may be used)

Q: WHAT IS THE TREATMENT OF ACUTE BRONCHOSPASM?


Treatment of acute Bronchospasm
During operation
1. Deepen the Anaesthesia.
2. Adrenaline 1:1000,0.5 ml sc.
3. Salbutamol infusion 5 mg/ml.
(5 ml of 1 mg/ml solution added to 500ml of 0.9% saline to give
final conc. Of 10mg/ml).
4. Ketamine, sub anaesthetic dose 0.75mg/kg.
5. Aminophylline 5mg/kg (halothane can interact with
Aminophylline to produce arrhythmias).

ANAESTHESIA AND LUNG DISEASES


Q: WHAT ARE THE COMMON LUNG DISEASES FACED DURING
ANAESTHESIA?
1. OBSTRUCTIVE LUNG DISEASE
A. Increased resistance to airflow can be caused by pathology
in the lumen, in the bronchial wall, or outside the airways.
Chronic obstructive lung disease
Chronic bronchitis
Emphysema
Reactive airway diseases (asthma)
2. RESTRICTIVE LUNG DISEASE
A. Pulmonary fibrosis
B. Sarcoidosis
C. Hypersensitivity Pneumonitis
D. Systemic Sclerosis

Q: WHAT WILL BE THE ANAESTHETIC MANAGEMENT OF A


PATIENT WITH LUNG DISEASE?
1. PREOPERATIVE EVALUTAION
a) Pulmonary function tests
b) ABGs
c) Chest X-ray

99

2. PREOPERATIVE PREPARATION
a) Minimizing bronchospasm
b) Smoking cessation at least 2 weeks prior to surgery.

3. ANAESTHETIC MANAGEMENT
a) Intubation
1) Direct oral: appropriate if intubation difficulty not
suspected.
2) Fibroptic: appropriate for difficult airway
3) Nasal
4) Awake: indications include difficult airway, high risk of
gastric aspiration, anatomic risk for inadequate cord
visualization, jaw malformation, head and neck scar,
congenital abnormalities of upper airway, morbid obesity
5) Endobronchial: appropriate when one lung ventilation
desired
b) Hypercarbia
1) May prolong time for resumption of spontaneous
ventilation at the end of case
2) Increases cerebral blood flow
c) Hypocarbia
1) May prolong time for resumption of spontaneous
ventilation at the end of case
2) Decreases cerebral blood flow
d) Hypoxemia: greatest risk
1) Low F1O2 due to anaesthetic circuit leakage or
disconnection
2) Endobronchial intubation
3) Bronchospasm
4) Pulmonary odema
5) Pneumothorax
6) Unplanned extubation
7) Endotracheal tube obstruction
8) Airway obstruction in nonintubated patient
9) Alveolar hypoventilation
10)
Atelactasis
11)
Worsening of underlying pulmonary disease

Q: WHAT ARE THE MOST COMMON POSTOPERATIVE


COMPLICATIONS?
1. Atelactasis
2. Pneumonia

100

101

Lecture 11

CARDIOPULMONORY RESUSCITATION
Q: DEFINE CPR.
Cardiopulmonary resuscitation (CPR) is an emergency technique that
anyone can learn to help someone whose heart and/or breathing has
stopped.

Q: What are the basic rhythms of Cardiac Arrest?


Rhythms of cardiac arrest are divided into two groups:
1. Shockable Rhythms
2. Nonshockable Rhythms
Each of these two are further divided into two rhythms
1. Shockable Rhythms:
a. Ventricular Fibrillation
b. Pulseless Ventricular Tachycardia
2. Non Schockable Rhythms:
a. Asystole
b. Pulseless Electrical Activity

Q: WHAT ARE THE POTENTIALLY REVERSIBLE CAUSES OF


CARDIAC ARREST.
The potentially reversible causes of cardiac arrest can be remembered
by keeping 5Hs and 5Ts.
1. 5Hs are as under:Hypovolemia.
Hypoxia.
Hydrogen Ion (Acidosis).
Hyper / Hypokalemia.
Hypothermia.
2. 5Ts are:Tablets (Drugs).
Temponade.
Tension pneumothorax.
Thrombosis, coronary.
Thrombosis, pulmonary.

102

Q: WHAT ARE THE TWO COMMONLY USED TYPES OF CPR.


1. Basic Life Support (BLS)
2. Advanced Life Support (ALS)

Q: DESCRIBE THE STEPS OF BLS.


Steps of BLS can be described in the form of Primary ABCD Survey.
Primary ABCD Survey is composed of following.
A Airway, Open the airway.
B Breathing,
Assess breathlessness
Give two normal breaths that make chest rise (1 sec)
C Circulation .. Assess circulation

Lay Rescuers .. No pulse check. Look for other signs of


circulation (breathing, coughing or movement).
Doctors and Paramedics .. Check pulse (Carotid in Adults,
Brachial in Infants and Children)
If no signs of circulation .. Start Chest Compressions

D Defibrillate. Attach AED or give shocks from manual defibrillator.


Single shock should be given. Monophasic 360 J and Biphasic 120200 J.

Q: WHAT ARE THE METHODS OF CLEARING THE AIRWAY?


There are two methods of opening the airway.
1. Head tilt and Chin lift Maneuver. This is done in patients where it
is confirmed that there is no risk of neck injury e.g. cardiac arrest
in home,
arrest in hospital wards.
2. Jaw Thrust Maneuver. This should be performed in patients in
whom neck injury is suspected e.g. road side accidents, drowning
in divers etc.

Q: HOW SHOULD ONE SUPPORT AND ASSESS BREATHING


DURING BLS?
103

Assess breathing by using three senses i.e. look, listen and feel. Look
for the movements of abdomen, listen the breath sounds, and feel the
air coming out of patients nose or mouth by placing your ear near
patients nose and mouth.
If the person is breathing place him in recovery position, call for help
and monitor his breathing efforts.
If the person is not breathing then give two normal breaths that make
chest rise, by blowing from your mouth after pinching his nose and
getting a proper mouth to mouth seal. After two initial breaths give two
breaths after every thirty chest compressions.

Q: DESCRIBE THE PROCEDURE OF CHEST COMPRESSIONS


DURING BLS.
Following are the essentials for chest compressions during CPR.
1. Hand placement. Place heal of your hands above the xiphoid
process in the midline of the sternum.
2. Compress the chest by two inches in adults, and one inch in
children (1 8 years).
3. Ratio of compression to ventilation (C:V) 30:2 in adults and
children C:V should be 30:2 for single rescuer and 15:2 for two
rescuers.
4. Compression rate should be 100/min.

Q: WHAT IS THE BEST TIME FOR DEFIBRILLATION?


Best time for defibrillation is immediately after cardiac arrest or as
early as possible. Chances of revival decrease by approximately 10%
after every minute delay in defibrillation.

Q: WHAT ARE THE COMPONENTS OF SECONDARY ABCD


SURVEY?
ALS can be described in the form of Secondary ABCD Survey.
Secondary ABCD Survey consists of following steps.

A Airway
Advanced airway control
Tracheal Intubation

104

B Breathing
IPPV with Ambu Bag or Ventilator

C Circulation
IV access
Rhythm appropriate drugs

D Differential Diagnosis

Q: WHAT AIRWAY ADJUNCTS COULD BE USED DURING ACLS?


1.
2.
3.
4.
5.
6.
7.
8.

Laerdal Mask (Ambu rescue mask)


Oropharyngeal airway
Ambu bag and mask
Laryngeal Mask Airway (LMA)
Oesophagotracheal Combitube
Endotracheal tube
Cricothyroid puncture
Tracheostomy (Percutaneous)

Q: ENUMERATE THE ESSENTIAL DRUGS REQUIRED FOR ACLS


ALONG WITH THE DOSES.
1.
2.
3.
4.
5.
6.
7.
8.

Oxygen 10 15 l/min.
Adrenaline 1mg every 3 5 minutes.
Amiodarone 300mg IV push in adults and 5mg/kg in children
Atropine 0.5 1mg (maximum dose is 0.03 to 0.04 mg/kg body
weight).
Lignocaine 1 1.5 mg/kg body weight.
Magnesium sulphate 1 2 g IV in adults.
Procainamide up to 17 mg/kg body weight.
Sodium Bicarbonate 1 mmol/kg only in protracted CPR, known
cases of bicarbonate responsive acidosis, hyperkalaemia.

Q: WHAT IS ALGORHITHM FOR BASIC LIFE SUPPORT?


105

ALGORHITHM FOR BASIC LIFE SUPPORT


Check responsiveness
Shake and shout

Unresponsive Shout
for help

Open airway, Head


tilt, Chin lift, jaw
thrust

Check breathing
Look, listen, feel

Breathing present, place in


recovery position

No breathing two
effective breaths

Assess circulation
Movement/pulse

Circulation present continue


rescue breathing @20/min,
check circulation every
minute

No circulation
Compress chest
100/min.30:2ratio

Q: WHAT IS THE ALGORITHM FOR CPR?


Cardiac arrest

106

BLS algorithm if
appropriate

Precordial thump if
appropriate

Attach defib/monitor

Assess
rhythm

VF/Pulseless VT

Defibrillate Shock
360J Monophasic
120-200J Biphesic

CPR 2 min

During CPR
If not already:
Check electrode/paddle
positions and contact
Attempt/verify: ETT
i.v.access
Give adrenaline every 3min
Correct reversible causes
Consider: buffers
antiarrhythmics
atropine/pacing
Potentially reversible causes:
Hypoxia
Hypovolaemia
Hyper/hypokalaemia and metabolic disorders
Hypothermia
Tension pneumothorax
Tamponade
Toxic/therapeutic disturbances
Thromboembolic/mechanical obsruction

Asystole or
PEA

Up to 3 min
CPR

Figure 1: Algorithm for advanced life support management.


BLS=Basic life support.

107

Fig: MOUTH TO MOUTH AND MOUTH TO NOSE VENTILATION. Start mouth to mouth, and if this
fails try mouth to nose. A, and B, extend the patients head, pinch his nose and watch his chest expand. B, and C,
when you ventilate mouth to nose, put one hand on his forehead and hold his chin up with the other one.

108

30:2

After every 30 chest compressions


2 lung inflations are carried out

Fig: CARDIOPULMONARY RESUSCITATION. Note that the operator is using the heel of his hand.

109

Q: HOW WOULD YOU KEEP THE AIRWAY CLEAR?


Try these methods in the following order. If one method fails, quickly
move on to the next.
FLEX THE PATIENTS NECK AND TILT HIS HEAD While he is lying flat,
grasp his head with one hand and tilt it so that his nostrils point
upwards. At the same time, flex his neck forward. Extending his head
without flexing his neck is less effective. This combination of
movements raises his mandible away from his cervical spine, and lifts
his tongue off the posterior wall of his pharynx. A pillow under his head
and neck helps to maintain this position.
LIFT HIS CHIN Pull it upwards. This will usually clear the airway of a
young adult with a good set of teeth.

Fig: TWO WAYS OF KEEPING A PATIENTS AIRWAY CLEAR. Tilting a patients head backwards will usually
clear his airway. If this does not, insert Guedels airway.

110

LIFT THE ANGLES OF HIS JAW Sit at the head of the table, rest your
elbows on it, and lift both the angles of his jaw with your middle
fingers. Your thumb and first fingers will then be free, if necessary, to
hold the mask, as in Fig.
Lifting his chin, if it succeeds, is better than lifting the angles of his jaw,
because lifting them can make his jaw stiff, and at worst dislocate it.
Lifting the angles of the jaw is for more difficult patients only.
GUEDELS AIRWAY If the above method fails to clear the patients
airway, insert Guedels airway.
Wet the airway. Open his mouth for a moment, and insert it with its tip
pointing towards his hard palate.
Then turn the airway through 180 so that its curve follows his soft
palate and the back of his tongue and lifts his tongue forward.
CAUTION! (1) Be careful not to push his tongue downwards as you
insert the airway. (2) Dont insert it during very light anaesthesia, or
the patient will cough, retch, or vomit. (3) Even Guedels airway does
not guarantee a clear airway, so you may also need to lift his chin or
the angles of his jaw.

Fig: GUEDELSS AIRWAY IN PLACE. If lifting a patients chin fails to clear his airway, you may need to lift the
angles of his jaw.

111

NASAL AIRWAY Put a soft wide rubber tube down one of his nostrils,
and hold it with a large safety pin. This is useful in severe maxillofacial
injuries, when opening the patients mouth may be impossible or
painful.
FERGUSSONS GAG is useful if the patient clenches his teeth shut, and
prevents you inserting an airway. Push the gag between his back teeth,
and use it to open his jaw. Keep pieces of rubber tube on the ends of
the gag to prevent them injuring his teeth. If his teeth are complete, so
that you cannot insert a gag, force a wedge between his teeth. Rock it
to and fro between them, until they are far enough apart for you to
insert the ends of the gag. The danger of this is that you will break his
teeth, but you may have to take this risk.
If you dont have a gag or a wedge, press your fingers between his
gums behind his molar teeth. This will open his jaws enough for you to
pass a laryngoscope. This is less traumatic than using a gag. Many
anaesthetists prefer this method and seldom, if ever, use a gag.

Q: WHAT IS THE NEW GUIDE LINE OF CPR, GIVE SUMMARY?


Summary of new ILCOR guidelines concerning BLS
Following the new recommendations of the Resuscitation council
2005, here are the main improvements to BLS some of which we might
try to get across during the WHO Earthquake training.
Main Changes
1. Children are classified into only 2 age groups: infants < 1year and
children (1year to puberty).
2. There are 5 initial rescue breaths (no mention that 2 must be
effective).
3. When doing pulse check it is now a circulation check: as follows:
Start chest compressions if any of the following are present:
Absent central pulse for up to 10 seconds (carotid, brachial or
femoral).
Inadequate pulse (< 60 bpm for all ages of Child) with signs of
poor perfusion (include pallor, lack of responsiveness and poor
muscle tone).
No signs of a circulation (include movement, coughing or normal
breathing but not agonal gasps).
4. Chest compressions must compress the lower third of the sternum
by one third of the anteroposterior diameter (use 2 fingers, 1 hand
or 2 hands as needed to achieve this depending on age and size).
5. Position for all ages is 2 finger breadth above the xiphisternum.

112

6. In infants hand encircling technique is best.


7. For all infants and children the ratio is 15 compressions 2
ventilations.
8. For all adults the ratio is 30 compressions to 2 ventilations.
Lecture 12

PAIN
Q: DEFINE PAIN.
Pain is a complex but an important protective phenomenon it may be
defines as:
An unpleasant sensory and emotional experience associated
with actual or potential tissue damage or described in terms of such
damage.

Q: WHAT ARE THE TYPES OF PAIN?


Pain is divided into two types, acute pain and chronic pain.
Acute pain: It is most often caused by an acute injury or
pathological state, and lasts only as long as the tissue lesion exists. If it
is not effectively treated, it may develop into chronic pain.
Postoperative pain is an example of acute pain.
Chronic pain: It is usually associated with a persistent tissue
lesion, it may continue long after the normal healing of an acute injury,
or disease has subsided. The term chronic is applied to pain which lasts
for more than six months.

Q: WHAT ARE THE PAIN PRODUCING SUBSTANCES?


PAIN PRODUCING SUBSTANCES:
A number of pain producing substances have been discovered
which are considered to be mediators of inflammatory reactions. These
mediators include the hydrogen and potassium ions, kinins, histamine,
5-hydroxytryptamine, metabolites of arachidonic acid, namely
prostaglandins and leukotrienes.

Q: WHAT ARE THE DIFFERENT METHODS FOR


MEASUREMENT OF PAIN?
Different methods used for measurement of pain include
1. Pakistan Coin Scale (PCS)
2. Verbal Rating Scales (VERSs)
113

3. Numerical Rating Scales (NRSs)


4. Visual Analogue Scales (VASs)
Theses simple methods have been used effectively in hospitals,
clinics and provide valuable information about pain and analgesia.
General principals / options of pain relief.
1. Prevent initial excitation of nociceptive nerves e.g., NSAIDs are
used, they inhibit prostaglandin activity.
2. Interrupt peripheral nociceptive transmission e.g., Local
anaesthetic in acute pain.
3. Alter spinal modulation of nociceptive transmission e.g.,
spinal/epidural
narcotics.
Epidural local anaesthetics.
Epidural local anaesthetic & narcotics.
TENS and Acupuncture.
There is inhibitory system in dorsal horn, TENS / acupuncture act
there.
Similarly spinal and epidural opiates act on receptors in dorsal
horn.
4. Interrupt spinal cord nociceptive transmission e.g., destroy or cut
the tract for intractable pain.
5. Alter central processing of nociceptive transmission e.g., opiates,
N2 O
etc.
They act centrally also.
6. Alter emotional response to pain e.g., there is anxiety in acute
pain and depression in chronic pain appropriate drugs can be
given.
7. Alter behavioral response to the pain e.g., in chronic pain patients
pain may submerge in a complete picture of disability.

Q: WHAT ARE THE COMMON DRUGS GIVEN FOR PAIN


RELIEF?
1. NSAIDs
Paracetamol
Aspirin
Diclofenac
Piroxicam
Mefenamic acid
Ketorolac
2. OPIODS
Morphine
Pethidine
Tramadol

114

Nalbuphine
Buprenorphine
Pentazocine

Q: WHAT ARE THE DIFFERENT BLOCKS GIVEN FOR PAIN


RELIEF?
Plexuses blocks
Brachial plexus block
Paravertebral block
Nerve blocks
Sciatic nerve block
Suprascapular nerve block
Ulnar nerve block
Radial nerve block
Peroneal nerve block
Biers block: IV Local anaesthetic is given after applying tourniquet to
an extremity.

Q: WHAT IS PATIENT CONTROLLED ANALGESIA(PCA) ?


This refers to the on-demand, intermittent, self- administration of
analgesic drugs by a patient. Predominately used to deliver opioid
analgesic. The traditional route of drug delivery has been intravenous
but subcutaneous and epidural routes can also be used. The quality of
analgesia is normally good and shows wide inter-patient variation. PCA
is also useful in children over 5 years, in obstetrics, in acute medical
diseases, e.g. sickle-cell crisis and malignant pain.

Q: HOW IS PAIN PERCEIVED?


Information signaling acute injury is transmitted along fast conduction
velocity (20m/sec) unmyelinated C fibers (dull second pain).
These first order neurons synapse with second order neurons in the
dorsal horn of the spinal cord. There is an increased release of
neuropeptides such as substance P, and excitatory amino acids such as
glutamate, in the dorsal horn. Nociceptive information to thalamus is
conducted up to the spinothalamic tracts. Synapse of second order
with third order neurons in the ventral thalamus results in onward
transmission of nociceptive impulse to sensory cortex, where it is
ultimately perceived as pain.

Q: WHAT ARE PAIN CLINICS?

115

Anaesthetists in conjunction with other specialties, including


rheumatology, neurology, neurosurgery, psychiatry and other
disciplines including nursing, usually head the pain clinics. The roles of
pain clinic in patient care are:
Decrease subjective pain experience.
Increase general level of activity.
Decrease drug consumption.
Return to employment or full quality of life.
Decrease further use of health care resources.
Essential pain clinic equipment will include suitable imaging,
suitable monitoring for sedation, radio-frequency lesion generator,
cryoprobe machine and peripheral nerve stimulator. Pain clinics
have been shown to be effective both for nerve block treatments
and for psychologically based therapeutics.

Q: HOW IS CANCER PATIENT TREATED?


Pain occurs in 70% of patient with advance cancer. Treatment of pain of
advanced cancer include treatments such as radiotherapy which is
effective for bony metastasis, chemotherapy, hormone manipulation,
orthopaedic correction of pathological fractures, surgical correction of
bowel obstruction and neurological decompression of cranium or spinal
cord. Dexamethasone is commonly used to reduce painful tissue
edema: WHOs step ladder analgesia are used.
When pain of advanced cancer is not adequately treated by active
treatment or weak opioids, the following options can be used.
Morphine, orally or an elixir.
Fentanyl patches with 72-h duration.
Diamorphine give subcutaneously.
Co-analgesics added to augment opioids such as NSAIDs,
tricyclic anti-depressants.
Epidural catheter analgesia using bupivacaine.
A number of neurolytic techniques are in common practice for
pain of advanced cancer.
Coeliac plexus block using alcohol or phenol for carcinoma of
pancreas, and pain arising from stomach, liver and small
intestine.
Splanchnic nerve blocks.
Chemical sympathectomy, lumbar or presacral for refractory
lower limb or pelvic pain.
Cordotomy done via radio-frequency of spinothalamic tract at C2
for mesothelioma.

116

Intrathecal neurolysis using phenol or alcohol for trunk pain.

Q: HOW PAIN IS RELIEVED DURING LABOUR. (PAINLESS


DELIVERY)?
Epidural catheter is passed during early stage of labour and local
anaesthetic drug is injected intermittently through the catheter.

117

Lecture 13

ICU
Q: DEFINE ICU.
An Intensive Care Unit usually provides 1 10% of total hospital beds,
apart
from
specialized
requirements
e.g.
cardiac
surgery,
neurosurgery, etc. Units larger than ten beds are sometimes
subdivided into specialized units, a maximum of four beds is
recommended.
Anaesthesiologists in the UK run 85% of ICUs.

Q: HOW IS AN ICU DESIGNED?


Design consideration include: Size of unit/bed (approx 20m2/ patient).
Proximity to OT, casualty department, X-Ray plus laboratory
facilities.
Equipment: ventilators, monitoring, infusion pump, etc.
Staff and their facilities (one nurse/patient for 24hr/day).

Q: WHAT IS THE PATIENT SELECTION CRITERIA IN AN ICU?


Patient selection criteria
Major organ failure requiring artificial support e.g., respiratory or
cardiovascular, often in combination with other organ system
failure (e.g. renal).
Intensive monitoring and treatment in severe disease states,
e.g., septicemia, head injury, poisoning and overdose, burns, etc.
Postoperative
monitoring
of
respiratory,
cardiovascular,
neurological problems, etc.
Other considerations the disease state should be potentially
reversible, pre morbid general health, age and mortality,
premedication scores and availability of beds.
Problems may be related to:
Original condition.
Multiple organ failure; may follow disease processes (e.g., renal
failure and ARDS are common in critical illness of any cause.
Prognosis worsens as more organ systems are involved.
Infection e.g., septicemia.
Adequate nutrition and fluid and electrolyte balance.
Gastric ulceration (stress ulcers).
Immobility: DVT and bed sores may occur.

118

Sedation.

MONITORING
Q: WHAT TYPE OF MONITORING IS REQUIRED IN ICU?
Monitoring required in an ICU include: Vital signs
ECG
CVP monitoring
ABGs
Pulse oximetry
Capnography
Pupil size and Urine output

ARTIFICIAL VENTILATION
Q: WHAT ARE THE MAIN TYPES OF ARTIFICIAL VENTILATION?
TYPES:1. Negative pressure ventilation (not used nowadays).
2. Positive pressure ventilation.

Q: CLASSIFY VENTILATORS.
Classification based on cycling from inspiration to expiration: Time cycled.
Volume cycled.
Pressure cycled.
Flow cycled.

Q: WHAT ARE THE INDICATION OF ARTIFICIAL VENTILATION?


Indications:
PaO2 < 60mm Hg.
PaCO2>50mm Hg.
PH<7.2
SPO2<80%
Respiratory rate > 35 or < 07
Unconscious patient not breathing.
Elective ventilation e.g. post thoracotomy, V/Q mismatch etc.

119

Q: WHAT IS THE CRITERIA FOR WEANING OFF THE


VENTILATOR?
Criteria for weaning off the ventilator:Patient should be weaned off early in the morning when all the
consultants are present.
PaO2 is more than 60 mmHg
PaCO2 is less than 50 mmHg
pH is greater than 7.2
SpO2 is greater than 80%
Respiratory rate < 35/min
Normal vital signs.

120

Lecture 14

COMPLICATIONS OF ANAESTHESIA
Q: WHAT ARE THE COMPLICATIONS OF ANAESTHESIA?

Respiratory complications.
Cardiovascular complications.
Complications resulting from posture.
Vomiting and regurgitation.
Neurological complications.
Awareness during Anaesthesia.
Malignant hyperpyrexia.
Accidental hypothermia.
Anaphylactic reactions.
Electrical hazards.
Miscellaneous complications.
Ophthalmologic complications.
Spontaneous rupture of tympanic membrane.
Minor sequelae.

Q: WHAT COMMON HUMAN ERRORS CAN LEAD TO


PREVENTABLE ANAESTHETIC ACCIDENTS?

Unrecognized breathing circuit disconnections.


Mistaken drug administration.
Airway management.
Anaesthesia machine misuse.
Fluid management.
Intravenous line disconnection.

Q: WHAT ARE THE RESPIRATORY COMPLICATIONS


ENCOUNTERED IN ANAESTHESIA?
Upper airway obstruction
Due to
Obstruction at the lips.
Obstruction by the tongue or soft palate.
Obstruction above the glottis, due to swab, tooth, foreign body,
saliva, vomitus, blood or oedema.

121

Obstruction at the glottis, due to laryngeal spasm, impaction of


epiglottis into the larynx, incomplete reversal of relaxants or foreign
bodies
Fault of apparatus

Bronchospasm
Due to

Asthma.
Surgical or airway stimulation.
Drug reaction.
Respiratory infection.
Pulmonary oedema.
Severe reduction in lung volume as in a tension Pneumothorax.

Coughing
Due to
Inadequate depth of Anaesthesia.
Chemical or infective inflammation of upper airways.
Irritation of larynx.

Hiccup

Due to
Stimulation of sensory nerve endings.

Sputum retention and atelectasis


Due to

Inadequate coughing due to pain.


Residual neuromuscular block.
Sedatives.
Impairment of mucociliary transport in lungs due to inhalation
of cold, dry gases.

Aspiration pneumonitis

Sleep apnoea
Due to
Airway obstruction.
Central respiratory depression (Ondines curse).
Pulmonary barotraumas
DVT and pulmonary embolism

Q: WHAT ARE THE COMMON CARDIOVASCULAR


COMPLICATIONS IN ANAESTHESIA?
Cardiac dysrhythmias
Associated with
Cardiac pathology.

122

Hypercapnia or hypoxia.
Toxins, malignant hyperpyrexia, drugs, anaphylaxis.
Electrolyte imbalance.

Myocardial ischemia and infarction


Stroke

Hypertension in immediate postoperative period


Possible causes are: Pain or full bladder.
Hypercapnia.
Confusion after anaesthesia.
Unsuspected pheochromocytoma.
Vasoconstriction after pulmonary bypass.
Thyroid crisis.
Air or gas embolism
Due to
Operations involving injury to veins in neck, thorax, breast and
pelvis.
Operations on brain and cord in sitting position.
Operations on the heart.
Laparoscopy.

Q: WHAT ARE THE COMPLICATIONS ASSOCIATED WITH


PATIENT POSITIONING?
Complications
Air embolism
Alopecia
Backache

Position
Sitting, prone, reverse
trendelenburg
Supine, lithotomy,
trendelenburg
Any

Compartment
syndrome

Especially lithotomy

Corneal abrasion
Digit amputation

Especially prone
Any

Nerve palsies
Brachial plexuses

Any

Common peroneal
Radial

Lithotomy, lateral
decubitus

Prevention
Maintain venous pressure
above 0 at the wound.
Normotension, padding, and
occasional head turning
Lumbar support, padding,
and Slight hip flexion
Maintain perfusion pressure
and avoid external
compression
Taping and lubricating eye
Check for protruding digits
before changing table
configuration
Avoid stretching or direct
compression at neck or axilla
Pad lateral aspect of upper
fibula

123

Any
Ulnar
Any
Retinal ischemia
Skin necrosis

Prone, Sitting
Any

Avoid compression of lateral


humerus
Padding at elbow, forearm
Supination
Avoid pressure on globe
Padding over bony
prominences

Q: WHAT ARE THE CLINICAL MANIFESTATIONS AND


TREATMENT OF ANAPHYLACTIC REACTIONS?
Clinical manifestations:Cardiovascular system
Hypotension
Tachycardia
Dysrythmias
Pulmonary
Bronchospasm
Cough
Dyspnoea
Pulmonary oedema
Laryngeal oedema
Hypoxia
Dermatologic
Urticaria
Facial oedema
Pruritus

Treatment:

Discontinue drug administration.


Administer 100% oxygen.
Epinephrine (0.01 0.5 mg IV or IM).
Intravenous fluids (1-2 L lactated Ringers injection).
Aminophylline (5-6 mg/kg IV).
Diphenhydramine (50-75 mg IV).

Q: WHAT ARE THE FACTORS CONTRIBUTING TO AWARENESS


DURING ANAESTHESIA?
Gas supply
Failure of N2O supply.
Gas leaks in the breathing circuit.
Failure to flush a circle system.
124

Vaporizer
Empty.
Not turned on.
Wrong agent in it.
Ventilators
Mixing of driving gas with respired gases.
Monitors
No agent monitor.
No N2O monitor.
No adequate monitor of awareness.
Technique
Insufficient sedation, premedication.
Total intravenous Anaesthesia.
Miscalculation of doses of intravenous drugs.
Difficult Intubation.
Patients
Resistance to Anaesthesia.
Alcoholism.
Very sick cases.
Emergencies.
Surgery
Obstetrics.
Cardiopulmonary bypass.
Bronchoscopy.

Q: WHAT ARE THE COMPLICATIONS OF ENDOTRACHEAL


INTUBATION?

Dental injury.
Trauma to pharynx.
Sore throat.
Transient hoarseness.
Arytenoids dislocation.
Vocal cord paralyses.
Disruption of laryngeal ability to protect the airway.
Persistent postoperative hoarseness.
Laryngeal injury.

Q: WHAT ARE THE REASONS FOR EMERGENCY INTUBATION


IN THE POSTANAESTHESIA CARE UNIT?
125

Airway obstruction related to head and neck pathology.


Bleeding at the surgical site.
Tracheal compression from a goiter.
Unexpected laryngeal oedema.
Laryngeal edema from prolonged upper airway surgery.
Post-tonsillectomy pulmonary oedema.
Excessive sedation or anaesthetic agent.
Inappropriate fluid management.
Persistent muscle relaxant effect.
Miscellaneous
(sepsis,
transfusion
reaction,
bronchospasm,
inappropriate intubation).

Q: DEFINE MALIGNANT HYPERPYREXIA AND HOW WILL YOU


MANAGE IT?
Definition
Malignant hyperpyrexia is an inherited autosomal disorder in
which heat production exceeds heat loss in the body to cause a rise of
temperature of at least 2C/h.
Management
Withdraw volatile anaesthetic agents and hyperventilate with
oxygen.
Cool patient with ice, wet sheets, fan, cold-water gastric and
peritoneal lavage.
Get help.
Measure the temperature, blood gases, and electrolytes.
Insert an arterial line, acidosis is corrected with bicarbonate.
Give Dantrolene 1-2 mg/kg, and repeat up to 10 mg/kg.
Hyperkalaemia can be corrected with glucose 50% (1liter) and
insulin 100 units. Promote a diuresis.
Give dexamethasone 20 mg, methylprednisolone 10 mg/kg or other
steroid.
Abandon the operation if possible.

126

127

Lecture 15

POST OPERATIVE RECOVERY AND CARE


Q: WHAT ARE THE COMPONENTS OF A POSTOPERATIVE
CARE UNIT ADMISSION REPORT?
Pre operative History:
Medication allergies or reactions
Underlying medical disease.
Chronic medications.
Acute problems (ischemia, acid-base, dehydration).
Premedication.
Vital signs.
Intraoperative factors

Surgical procedure.
Surgeon.
Type of anaesthesia.
Relaxant/reversal status.
Unexpected surgical or anaesthetic events.
Intraoperative vital sign ranges.
Estimated blood loss.
Urine output.
Drugs given.
Time/amount of opioids administration.

Assessment and report of current status

Heart rate and rhythm.


Systemic blood pressure.
Temperature.
Ventilatory adequacy.
Level of consciousness.
Endotracheal tube position.

Post operative instructions

Acceptable vital signs ranges.


Anticipated CVS problems.

128

Expected airway and ventilatory status.


Acceptable urine output and blood loss.
Surgical instructions.

Q: WHAT IS POTOPERATIVE RECOVERY SCORE?


SALIM ABC RECOVERY SCORE BASED ON PHYSICAL SIGNS
Airway
jaw

3
can cough or
Cry

2
maintains
airway without

1
holding of

0
holding of
and
other

jaw needed

measures
holding jaw

taken to

maintain
Behaviour can lift the head
movement at all

can open eyes

some non-

airway
no

and show tongue

purposeful
Movements
awake but needs
responds to

Consciousness fully awake,


no
response
Can talk, well
support
stimuli only
Oriented
A score of 8 is the minimal for discharge from the recovery.
Adapted from: Lees Synopsis of anaesthesia; 12ed. 1999 Page 62.
OTHER TESTS FOR RECOVERY ARE NOT RELIABLE e.g. : Critical flicker fusion threshold visual and auditory, a measure of
sensory impairment.
Picture recall.
Backward spelling of common 4-lettered words.

129

Q: HOW WOULD YOU DIFFERENTITATE PAIN, HYPERCAPNIA,


AND HYPOVOLEMIA IN RECOVERY ROOM?
Pain
May be restless
May be quiescent
if severe pain

Hypercapnia
Comatose

Periphery

Vasoconstriction,
Pallor sweating

Heart rate

Tachycardia

Warm, flushed
with bounding
pulse
Tachycardia

Arterial pressure

Systolic

Systolic

Diastolic

Diastolic

Pulse pressure
normal

Pulse pressure

Conscious level

Hypovolaemia
Restless or
quiescent
depending on
extent of
analgesia and
residual
anaesthesia
Vasoconstriction
, pallor
sweating
Tachycardia
Systolic and
diastolic may be
normal until
marked
reduction in
stroke volume
then
Pulse pressure

Pain causes restlessness and hypoxia also causes restlessness.


Hypoxia + analgesia = death.
Hypoxia + O2 = OK.
So in pain give analgesics and in restlessness due to hypoxia give
O2.

Q: WHAT IS THE DIFFERENTIAL DIAGNOSIS OF FAILURE TO


REGAIN CONSCIOUSNESS AFTER ANAESTHESIA?

Prolonged drug reaction


Overdose.
Increased sensitivity.

130

Decreased protein binding.


Redistribution.
Drug interaction.

Metabolic
Hypoxia or Hypercapnia.
Hepatic, renal, or endocrine end organ dysfunction.
Hypoglycemia, hyperosmolar hyperglycemia, diabetic
ketoacidosis.
Electrolyte imbalance (Na+, Ca++, Mg+).
Neurologic injury
Intracranial hemorrhage.
Cerebral ischemia.
Cerebral embolus.
Cerebral contusion.
Subclinical seizures.
Enlarging pneumoencephalus.

Q: WHAT ARE THE CAUSES OF POSTOPERATIVE SINUS


TACHYCARDIA?
Drug
Withdrawal (beta blockers, clonidine)
Theophylline
Atropine
Beta-2 antagonists
Epinephrine
Vasodilators
Ketamine
Others
Hypovolemia, hypervolemia
Anemia
Fever
Hypercarbia
Hypoxemia (early sign)
Cardiac tamponade
Tension pneumothorax
Thromboembolism
Hyperthyroid, pheochromocytoma
Pain
Anxiety, Shivering
131

Bladder distention
Allergy
Hypoglycemia
Acute porphyria

Q: WHAT ARE THE COMMON CAUSES OF POSTOPERATIVE


HYPERTENSION?

Preexisting hypertension.
Withdrawal of antihypertensive medications.
Hypercarbia.
Volume overload.
Bladder distention.
Increased intracranial pressure.
Pain.
Drugs.
Pressors, epinephrine, Ketamine.
Reversal with naloxone.
Indirect acting vasopressor with chronic MAO use.
Autonomic hyperreflexia.
Pheochromocytoma.
Carotid surgery.

Q: WHAT ARE THE COMMON CAUSES OF POST OPERATIVE


HYPOVENTILATION?
Factors affecting
airway

Factors affecting
ventilatory drive

Peripheral factors

Upper airway
obstruction

Respiratory

Muscle weakness:

Tongue

Preoperative or

Laryngospasm

Oedema
Foreign body
Tumours
Bronchospasm

depressant
Postoperative CNS
depressant drugs,
Cerebrovascular
vascular accident

Hypothermia
Recent

Residual
neuromuscular block
Preoperative

neuromuscular
disease

Electrolyte

abnormalities

132

hyperventilation
(PaCO2 low)

Pain
Abdominal distention
Obesity
Tight dressings
Pneumo-/haemothor
ax

133

Fig: A, THE RECOVERY POSITION is the only safe one for a patient on the trolley on his way to the ward, and in
his bed when he gets there. Show your nurses how to place an unconscious patient on his side, with his uppermost arm
and leg supporting his body. This position helps to keep his airway clear, it allows his tongue to fall forwards, and it lets
blood and secretions drain from his mouth. B, sucking out his nose. Pinch one of this nostrils shut while you suck
through the other.

134

Fig: ALL SET FOR A SAFE RECOVERY on a trolley which has sides and can tip. There is an oxygen cylinder and a
mask, a bell to summon help, a sphygmomanometer, and a sucker.

135

Q: WHAT IS THE CLINICAL ASSESSMENT OF THE ADEQUACY


OF REVERSAL OF NEUROMUSCULAR BLOCK?
Subjective:Grip strength
Adequate cough
Objective:Ability to sustain head lift for at least 5 sec.
Ability to produce vital capacity of at least 10 ml/kg

Q: WHAT ARE THE COMMON CAUSES OF PERIOPERATIVE


FEVER?

Infections
Immunologically mediated processes
Drug reactions
Blood reactions
Tissue destruction (rejection)
Connective tissue disorders
Granulomatous disorders
Tissue damage
Trauma
Infarction
Thrombosis
Neoplastic disorders
Metabolic disorders
Thyroid storm (thyroid crisis)
Adrenal crisis
Pheochromocytoma
Malignant hyperthermia
Acute gout
Acute porphyria

136

APPENDIX
GLASGOW COMA SCALE
Eyes open
Spontaneously
To command
To pain
No response

4
3
2
1

Motor response
Obeys command
Localizes pain
Withdraws
Flexion (abnormal)
Extension (abnormal)
No response

6
5
4
3
2
1

Verbal response
Oriented
Confused
Inappropriate words
Incomprehensive sounds
No response

5
4
3
2
1

Scores <7 are defined as coma.

QUICK GUIDE TO EVALUATE A CHEST FILM


Structure
Trachea
Clavicles
Hilum
Heart
Lungs
Diaphragm
Costophrenic angles

Normal
Midline position
Equidistant from sternum
White densities where bronchi join lungs; left
hilum is 23 cm higher than right hilum
Cardiothoracic ratio < 50%
Radiolucent
Right side is 1-2 cm higher than left; should be
rounded structures.
Clear and sharp

ABNORMAL RADIOGRAPHIC FINDINGS


137

Findings

Possible Diagnosis

Non-distinct or widened aortic knob


Aortic dissection
Silhouette sign (loss of border visibility) Infiltrates or consolidation
RML or
lingula
Blackened area
Pneumothorax
Patchy infiltrates or streaky densities
Pneumonia, atelectasis
Fluffy infiltrates (Kerly B lines)
Pulmonary edema
Loss of Costophrenic angle sharpness
Pleural effusion

INTUBATION/EXTUBATION GUIDELINES
Intubation:

Tube size:Orotracheal
Nasotracheal
Males: 8-8.5 mm i.d.
<7.5 mm i.d.
Females: 7-8 mm i.d.
Cuff pressure:>20 mm Hg increases risk for tracheal damage
<15 mm Hg increase risk of aspiration around cuff
Ventilation:Auscultate the lateral aspect of the chest midaxillary line for
presence of breath sounds.
Inspect chest for equal expansion.
Auscultate over the epigastric area. Gurgling sounds indicate
esophageal intubation
Minimal occlusive technique:Place stethoscope at larynx.
Slowly remove air (in 0.2 ml amounts) from cuff until air leak is
heard.
Slowly reinsert air (in 0.2 ml amounts) until the inspiratory leak
stops.
Stabilize tube:Remember :- Regarding tube when in doubt take it out.

Extubation:

Elevate HOB. (Head of Bed).


Preoxygenate (100%).
Suction endotracheal tube, above tube cuff, from patients mouth.

138

Instruct patient to take in a deep breath; deflate cuff and remove


tube on peak inspiration.
Administer prescribed oxygen.
Assess signs and symptoms indicative of respiratory distress and
increased effort.

Signs and Symptoms Indicative of Respiratory


Distress and Increased Effort;
If pulse < 60/min or > 120/min
If BP

or

by 20 mm Hg of base line

If RR >30 or <8
If PAWP > 20mm Hg
Other findings: dyspnoea, panic, fatigue, cyanosis, dysrhythmias, nasal
flaring, intercostals retractions, altered breathing pattern, paradoxical
motion of rib cage and abdomen.

CRITERIA FOR WEANING:


Parameter and normal value
weaning

Criteria for

Vital capacity (65-75 ml/kg)


Tidal volume (5-7 ml/kg)
Respiratory rate (10-12 breaths/min)
Minute ventilation (5-6L/min)
Negative inspiratory force (-75 to 100 cm H2O)
PaCO2 (35-45 mm Hg)

>10-15
>5
12-20
<10
>-25
<45 (except
COPD patients)
>70 on FiO2 40%
>16
<0.6

PaO2 (80-100 mm Hg)


FEV1 (50-60 ml/kg)
VD/VT (<0.3)

On Arrival of Patient in Operating Room:


1.
2.
3.
4.
5.

Verify name of patient, site of operation and consent form.


Set up IV drip and monitors.
Pre Oxygenate the patient after re assurance.
Give induction agents and other drugs through IV line.
Pass endotracheal tube where necessary.

139

6. Give O2 + N2O + Volatile agents through mask or endotracheal tube


through out the surgical procedure, the muscle relaxant are given if
necessary.
7. At the end of operation muscle relaxants are reversed with
neostigmine and atropine/pyrrolate.
8. Patient is sent to recovery room for further care.

MAINTENANCE OF FLUID REQUIREMENTS


These are based on body weight: For the first 10kg 100ml/kg/day (or
approximately 4 ml/kg/hr). For the next 10kg 2ml/kg/hr). Remember,
infants need 4 ml/kg/hr of fluid in and at least 2 ml/kg/hr of urine
output. Bigger children need 2 ml/kg/hr in and 1 ml/kg/hr out.

INCREASE FLUID REQUIREMENTS


Remember the mnemonic A, B, C, D, E, F:
Air: intubated patients or those with tracheostomies who breathe
unhumidified air.
Burn: fluid loss through a burn or open wound can be several times
the maintenance.
Cavities: Places to hide water that make it less available to the
vascular space such as the bowel with obstruction of
retroperitoneum with pancreatitis.
Diarrhoea.
Enterocutaneous fistulas.
Fever: 10% increase for degree F increase in body temperature.

TO CORRECT Na+ DEFICIT SAFELY


The amount of Na= required to increase the serum Na + to a desired
level can be calculated: Na+ needed (in mEq) = (target Na + - actual
Na+) 0.6 (wt. in kg). The amount of Na + needed to be infused over a
certain time frame to raise the Na+ 0.5 mEq/hr can thus be calculated.

SOME CAUSES OF METABOLIC ACIDOSIS WITH AN


INCREASED ANION GAP
Use the mnemonic MUDPILES:
Methanol intoxication
Uremia
Diabetic ketoacidosis
Paraldehyde intoxication
Infection
140

Lactic acidosis
Ethylene glycol, Ethanol intoxication
Salicylate intoxication

THE DISTRIBUTION OF TOTAL BODY WATER (TBW) IN


AN ADULT
TBW = 60% of total body weight; extracellular = 20% of total body
weight (15% interstitial + 5% intravascular);
Intracellular = 40 % of total body weight.

THE Ws OF POSTOPERATIVE FEVER


1. WIND: atelectasis usually first 24-48 hours after surgery.
2. WEIN: IV catheter classic third day fever.
3. WATER: urinary tract infection - usually occurs 5-8 days after
Foleys catheter inserted.
4. WOUND: usually 6-10 days after surgery.
5. WHERE: positive intra-abdominal abscesses usually 7-14 days
after surgery.
6. WALKING: thrombophlebitis in lower extremities usually 7-14
days after surgery.
7. WONDER : drug fever (wonder drugs antibiotics?).

Training Procedures for Students/House Surgeons in Operating


Rooms
1. Pre operative assessment.
2. Venepuncture.
3. Maintenance of airway with mask.
4. Intubations.
5. LMA insertions.
6. Maintenance of anaesthetic record.
7. arterial puncture.
8. ABGs.
9. Pharmacology of anaesthetic drugs.
10. Local Blocks.

141

Fig: THE TEN GOLDEN RULES, If these rules were always followed, there would be far fewer anaesthetic deaths:
(1) Assess and prepare a patient adequately. (2) Starve him. (3) Put him on a tipping table. (4) Check the machine and
cylinders before you start. (5) Have a sucker ready. (6) Have airways ready. (7) Be ready to control his ventilation. (8)
Have a vein open. (9) Monitor his pulse and blood pressure. (10) Have someone around who can apply cricoid
pressure, and who can be relied on in an emergency.

142

LOG BOOK
PROCEDURES
1. Pre operative assessment of the patient.
2. I/V Cannulation and Intraoperative fluid Management.
3. Induction of General Anaesthesia and Tracheal Intubation.
4. Demonstration of Spinal Block.
5. Demonstration of Epidural Block.
6. Demonstration of Local Blocks in Eye, E.N.T and General Surgery.
7. Demonstration of C.P.R.
8. Post Operative Care / Pain Management.
9. Introduction to the I.C.U.
10.

Demonstration of Anaesthesia Machine and other instruments.

11.

Demonstration of Sterilization procedures in O.T and I,C.U.

12.

Demonstration of Vital Sign Monitors and their application.

SPECIMAN OF STUDENT CARD WHICH SHOULD BE


MAINTAINED DURING TRAINING.

143

DEPARTMENT OF ANAESTHESIOLOGY
NAME OF THE HOSPITAL
____________________________________________________________
____________________________________________________________
Name:

____________________________________________

Roll No:

____________________________________________

Year: _________________________________________________
_______________________________________________________
G.A. OBSERVED (No of Cases)
Spinal/A. OBSERVED (No of Cases)
VIVA

__________
__________
__________

ATTENDANCE

__________

OVERALL PERFORMANCE GRADE

__________

____________________________
SIGN HEAD OF
DEPARTMENT

144

Key

Viva

A
B

Excellent
Good

Satisfactory

Poor

PERFORMED
PROCEDURE

Attendance

80%
7080%
6570%
60%

>75%
60-75%
50-60%
50%

DATES WITH INITIALS

Pre-Operative assessment

Venepunctures

Maintenance of airway with


Mask

Laryngoscopy & Intubations

LMA Insertion

Spinal Block

Machine & Monitoring

Fluid Resuscitation

CPR (Basics)

____________________________

SIGN HEAD OF
DEPARTMENT
ATTENDANCE RECORD OF 15 DAYS
DATE

SIGN

DATE

SIGN

145

_______________
Total Attendance

________________________
Sign Head of Department

146

SUGGESTED READING
Students those who are interested to become anaesthetist and planning for postgraduation in anaesthesiology are suggested to read the following books:1. Morgans Text Book of Anaesthesia
2. Aitkenhead Text Book of Anaesthesia
3. Lee Synopsis of Anaesthesia
4. Problem Orientated of Anaesthesia
5. T-E-OH of Intensive Care
6. Anaesthesia Secrets
7. Millers Anaesthesia
8. ParBrooks Basic Physics for Anaesthetist
9. Journals: i) British Journal of Anaesthesia (BJA)
ii) Anaesthesia
iii) Anaesthesia and Analgesia

147

Vous aimerez peut-être aussi